Categories
Funny Business M.I.T.

M.I.T. Dystopian Faculty Skit by Solow,1969

 

 

The current events of the late ‘sixties are the clear inspiration for this somewhat dark, dystopian skit for the M.I.T. economics departmental Christmas party of December 1969. According to the cover page, it was written by Robert Solow with input from Frank Fisher.

The skit was transcribed from the typed text [that includes a short handwritten addition] from Robert Solow’s papers in the Economists’ Papers Archive at Duke University. A grateful tip of the hat to Roger Backhouse for this artifact that should keep a cultural historian of economics busy for a few hours and be worth a few minutes of procrastination for working economists.

 

Pro-tip: you can summon all of the Economics in the Rear-view Mirror posts with economic humor content using the keyword “Funny Business”:

https://www.irwincollier.com/category/funny-business/

_______________________

Back-story for selected references in the text

SPECTRE. In Ian Fleming’s world of James Bond the acronym for the organization of international evil [Special Executive for Counter-intelligence, Terrorism, Revenge and Extortion].

Chairman Edel. Assistant Professor Matthew D. Edel (Yale, Ph.D.) taught the course Economic Growth and Development. Presumably pronounced to rhyme with “Fidel”. Edel was a regional expert for Latin America, spoke at a colloquium February 4, 1970 on “The Strategy of Cuban Economic Development

14.463 Monetary Economics in term I, 1969-70 was taught by four instructors.

According to the staffing report for that term in the departmental records at the MIT archive.

Karen H. Johnson, M.I.T. Ph.D. (1973),
Robert K. Merton, M.I.T. Ph.D. (1970), advisor Paul Samuelson
David T. Scheffman, M.I.T. Ph.D. (1971), advisor Paul Samuelson
Jeremy J. Siegel, M.I.T. Ph.D. (1971)

There is no record that Bonnie Parker and Clyde Barrow were ever graduate students of economics in M.I.T.

Bread and Roses. Reference to the Women’s Liberation Organization in Boston, 1969-1971. The name chosen in memory of the Great Lawrence Strike of 1912.

Ted Behr. An M.I.T. Ph.D. (1969) who by 2009 had already gone through seven career changes and twelve jobs. Must have been quite a character judging from this interview.

I think we may assume that no Bulgarians were injured in the writing or performance of this skit.

_______________________

Some Obvious Context

Fall 1964. Berkeley Free Speech Movement

Wikipedia Entry on the Protest Year 1968

April 1968. Columbia Student Strike ; Harvard Student Strike

February 1969. Black student strike at the University of Wisconsin

_______________________

RIP VAN SAMUELSON RETURNS TO MIT AFTER THE REVOLUTION
FACULTY SKIT
Christmas 1969

CAST

P. Diamond
R. Eckaus
R. Engle
F. Fisher
C. Kindleberger
M. Piore

SCRIPTWRITER-IN-CHIEF — R. Solow

HELPED BY – F. Fisher

Is it really true that Samuelson has been asleep all these years? Then how come the 13th and 14th editions of the textbook came out on time?

Well, I don’t know. Samuelson isn’t talking.

Careful, there. If it’s not talking it’s not Samuelson.

It’s got to be. His broker recognizes his fingerprints from soiled sell orders. Actually, there are two schools of thought about how the textbook came out while Samuelson was sleeping. Modigliani claims that the 13th and 14th editions were simply forecasted by the FRB-MIT model, using a long lag. But some people believe that the 13th and 14th editions are just the 2nd and 3rd editions reprinted. Can’t verify that, though. Nobody’s been able to find a copy of the early editions.

Not that it matters. Must be a shock for Paul to realize that nobody uses the text any more, except of course for the Bulgarian translation. They’re the only people reactionary enough to go for that stuff any more.

You mean even Hanoi University has dropped it?

Oh sure, they adopted Best Known Thoughts of Chairman Edel, last year. You know, the one that begins “Equilibrium grows out of a barrel…”

Out of the barrel of a gun?

No, no, a barrel of rum. Chairman Edel never got over that trip to Cuba.

Did you fellows hear that Samuelson is back? When did he disappear anyway?

Oh, a long time ago. Even before Chomsky became President. It’s hard to know the exact date. Things were pretty clear up until April 1972, when we were supposed to have 31 days of moratorium, but the month only had 30 days, so we cancelled the first day of May, only you couldn’t cancel May Day — Christmas you could cancel, but not May Day. So we cancelled the second day of May. But then we were three days short to fit in the 32 days of moratorium for that month, so we had to run into June. From then on it was chaos.

Things are still a little funny. I can’t get used to having summer vacation in the middle of winter, and Fisher pretending to go off skiing when it’s 90 degrees in the shade, when we all know he’s leading rent strikes anyway.

Don’t complain. It might have been worse. Solow claimed to have a proof that the term would never end once we got up to 32 moratorium days a month. But one of the younger mathematical economists made a brilliant application of the theory of Riemann surfaces and showed that you could pack any finite number of moratorium days into one month if you did it right.

It was the last article anyone published in this department. Can you remember when we used to write articles and hope for tenure? That was before tenure was abolished. God, life was easy then. Nowadays it’s all action, action, action. And if you’re lucky, if you happen to win a rent strike, or destroy some draft records, or win an amateur topless contest, then maybe the central committee of SPECTRE will keep you on for a year. But suppose you lose the strike, or you let a white man go to work on a construction site, boy that SPECTRE can be tough. You remember when they threw Domar into the arena with Kampf and gave Kampf the bullhorn?

I looked away. Bloodthirsty crew — they awarded Kampf both ears and the tail that day. We had to take up a collection to send Ricky and Alice [note: Evsey Domar’s daughters] to Bread and Roses Karate School. And today they’re members of SPECTRE, the Student Power Electoral Committee for Teachers of Relevant Economics. It was better in the old days when appointments went on good looks and amiability. Even publishing was better than action all the time. That last piece of work I did, keeping the recruiter for Mars Bars from getting onto the campus, it went well but it was exhausting.

Why are we against Mars Bars?

Space, military, it’s all the same.

Anyhow, now that he’s back, what’s Paul going to do around the department? He’s getting a little old for real action, and he might find it hard to pass the monthly Relevance Check.

It’s going to be a problem. He was falling behind the times when he went to sleep. Of course he looks better now, with 10-15 years growth of beard, but he doesn’t dig the revolution. El Lider Maximo of the Graduate Student Commune asked him what he could contribute, and Samuelson said he’d like to teach the History of Economic Thought.

The History of WHAT???

That’s exactly what the Commune Lider said.

Poor old Samuelson doesn’t know that Thought isn’t Relevant. In fact he didn’t even know that Economics isn’t Relevant. When El Lider explained that it was all action now, old Samuelson said he thought there should be both Thought and Action just so their marginal net productivities were equal.

Gad, I haven’t heard anything like that since the day they fired Diamond for saying “Pareto-optimal” once too often.

Whatever happened to Diamond?

What else, he’s at B.I.T., the Bulgarian Institute of Technology. Boy, if the old stuff ever comes back in style, those Bulgarians will have it made. But go on, what happened when Samuelson pulled that bourgeois bit about marginal whatnots?

Well, Solow was standing there and he muttered something to Samuelson—it sounded like “Check the second-order conditions, Paul old boy”—and then went back to trying to look hip.

That’s living dangerously.  Solow just barely passed last month’s Relevance Check, and he hasn’t been on a successful action in a long time. I don’t think that went over so good when he claimed that skiing Black Mountain was a real action. He better watch out — if B.I.T. won’t take an old man like that, SPECTRE may throw him to Kampf.

Right on. Nothing gets past El Lider. When Solow whispered that to Samuelson about second-order conditions, El Lider asked him right away — Did you say something? Solow replied Negative. Definite. That’s really living dangerously — I think it’s code of some kind.

It certainly doesn’t sound Relevant. I haven’t read anything like that in Ted Behr’s Newsweek column, at least not lately.

What’s going on this week in the department?

In the Theory course we’re holding an obstructive picket line at the drug counter of the Tech Store. Somebody discovered they were selling only white pills.

If I know what the pills are for, I hope the picket line isn’t too obstructive.

Of course not; I told you it was the Theory course. Then in the Economics of Education course we’re going to burn down a school. In the Money course, Johnson, Merton, Siegel, Bonnie, and Clyde are going to rob a bank and distribute the proceeds to the C.L.F.

Is that the California Liberation front?

Oh no, Berkeley has been a free-fire zone for months; nobody is left. It’s the Center for Love and Finance, our answer to the profit motive. Has anyone told you what the Econometrics Commune is doing?

No. Last week somebody had an idea for an empirical paper, but the results only came out at the 10% Relevance Level and half the commune was purged for Type One Error.

Served them right. Any Type II Error executions?

You know we have to have public trials for Type II error.

That’s right—Power to the People…. Well, it’s nice to see that the action curriculum is moving along. Sure beats the Old Days before chairman Edel — remember when they taught about Indifference curves? INDIFFERENCE curves, mind you, with innocent people being napalmed in Laos, Birmingham, Princeton, they taught about indifference curves.

Hard to believe. Of course now, ever since we adopted Bohmer’s best-selling text Economics for Good Guys we handle all that stuff by the tangency of the Relevance Map and the Isoconcern lines. Makes all the difference in the world, takes the subject out of the mind and puts it back in the gut, where it obviously belongs.

The Admissions Commune has been meeting all day.

How does the entering Movement look?

Terrific. There’s one girl who was heavyweight sugar-cane-cutting champion of the Big Ten, and another who had already led three successful rent strikes as a junior — two of them publishable, according to her advisor. Then there are a couple of Black Belts from Bread and Roses — they come on Karate Scholarships of course.

Any amateur topless contest winners?

We’re trying for a few, but most of them will go to Harvard—ever since they hired Brigitte Bardot for the economics faculty—

She was past her peak.

Peaks. And aren’t they all? Anyhow, all the amateur topless winners go to Harvard. But we’ve got some applicants who’ve starred in home movies. Not to mention a few school-burners and a couple of guys who have specialized in destroying computers.

How are their vibrations?

Good.

Fine. If there’s anything I can’t stand it’s bad vibrations. How about GRE scores.

The Graduate Relevance Exam grades just came — most of the people we’re accepting are in the 800’s on Obstructive and at least 750 in Vituperative. Looks like a good class — I mean Movement.

Has anyone heard what the Placement and Appointments Committees have decided?

They decided to eliminate the middleman and merge. That way everybody stays forever — once a Commune always a Commune. It gives new meaning to that old phrase about departmental inbreeding.

We still have this problem about what to do with Samuelson. Here he is after all those years asleep and hardly knowing anything about action and relevance and all the new things. The Bulgarians won’t take him — B.I.T. doesn’t mind using the old textbook, but they’re overloaded with these old-timers. If we can’t find something for him to do we may have to throw him to….

Terrible news. The students are revolting again. There’s a new movement sweeping all the Communes. They want one day of classes this month, two days of classes next month, three days the month after…there’s no telling where it will end, except that nobody can count over 30 any more.

Gad, we may have to go back to teaching again. Well, at least that gives something for Samuelson to do.

Oh didn’t they tell you. When Samuelson saw what the new system was like, he went back to sleep. Better get the Bulgarians on the phone.

 

Source: Duke University. David M. Rubenstein Rare Book and Manuscript Library. Economists’ Papers Archive. Papers of Robert M. Solow, Box 83.

Image Source: Robert Solow in his office, MIT Museum Website.

Categories
Exam Questions Fields M.I.T.

M.I.T. General exams for international economics, 1959

 

It seems safe to assume that Charles Kindleberger was the principal author of these general exams for the field of international economics (i.e. international trade and finance) since the exams come from his papers at the M.I.T. archive. I don’t know whether he had been the sole author. Maybe Samuelson contributed an international trade question or two, but that is much more speculative than Kindleberger’s likely authorship.

The general exams in international economics for 1950-51 have been transcribed and posted earlier.

_____________________

INTERNATIONAL ECONOMICS
February 16, 1959

Part I

Write an essay on any two or three of the following topics.

  1. The gains from trade.
  2. The effect of foreign trade on the distribution of income.
  3. Structural disequilibrium in the balance of payments.
  4. What determines the commodities and services a country will export and import?
  5. Elasticity conditions in international trade.

Part II

Answer any two or three of the following questions.

  1. A distinguished economist has stated that an underdeveloped country which is not developing balance of payments trouble, is not trying very hard to develop. Explain this view and discuss it critically.
  2. The New York Times recently had an article explaining that the present favorable position of the British and other West European balances of payments was really a bad sign because it was accompanied by a reduction in the volume of world trade. In particular, the improvement in the British balance of payments was due to a sharp improvement in the terms of trade which could not help worsen the situation after a few months.
    What have favorable or unfavorable terms of trade to do with the matter?
  3. What is the purpose of two of the following. How well have they filled, or are they filling, that purpose?
    1. The International Bank for Reconstruction and Development.
    2. The International Monetary Fund.
    3. The Colombo Plan.
    4. The Marshall Plan.
    5. The European Payments Union.
  4. The following is a real quotation from a distinguished economist: “Under a system of free trade there would be conflicts in interests neither among different nations nor among corresponding classes of different nations.”
    Discuss critically.
  5. “If all countries pursued full employment policies and at the same time avoided inflationary pressures, the balance of payments would present no problem.” Discuss theoretically.
  6. “There is no reason why a country could not pursue any domestic policy it liked provided it did not care about exchange stability.”
    “A country could have any fixed exchange rate it chose provided it pursued the correct domestic policy.”
    Discuss these quotations critically.
  7. Write an essay on the advantages and disadvantages of aiding underdeveloped countries through private capital movements, governmental loans and gifts on a bilateral basis or through multilateral aid.
  8. “It is frequently stated that aid should be given ‘with no strings attached.’ And this is a meaningless statement, because you can’t just send an anonymous check and say: do what you want.”
    What is meant by such a statement? What conditions could be attached to aid to make it effective?
  9. Write an essay on the instruments of commercial policy and discuss the effectiveness of each.

 

*  * *  *  * *  *  *  *

General Examination in International Economics
May 20, 1959

Answer any five questions

  1. Discuss the relevance of the factor-price equalization theorem to the observed facts of international trade.
  2. It has been said that the theory of international trade is peculiarly static and that this vitiates its applicability to the problems of growing economies. Do you agree or disagree? Discuss.
  3. Analyze the relevance of international trade (and tariffs) to wages and employment in as many contexts as are significant.
  4. What differences exist between internal trade in a single country, economic integration between sovereign countries, and international trade between unintegrated countries? Is there more content to economic integration than customs union?
  5. Discuss the relative roles of income and price in international adjustment, not in theoretical models, but as they have operated in the real world as observed by historians, by econometricians, or by casual empiricists. What generalizations, if any, can be drawn from this experience regarding the efficacy of exchange depreciation in producing adjustment?
  6. Argue for or against central bank intervention in the forward exchange market.
  7. What can the economist say about foreign aid?
  8. Compare and contrast the impact of foreign trade and lending on economic stability in a developed country and in an export economy? What monetary and commercial policy devices are available to the latter to promote stability?
  9. Write brief didactic essays setting forth the “correct view” (conventional wisdom) of international trade economists on two of the following subjects:
    1. the long-run terms of trade facing underdeveloped countries;
    2. the persistent surplus in the German balance of payments;
    3. the regional vs the universal approach to commercial policy and intergovernmental lending;
    4. commodity price stabilization;
    5. multiple exchange rates: blessing, menace, crutch for the feeble?
  10. Argue the case for modifying the Articles of Agreement of the International Monetary Fund, or its procedures under the present articles, or for leaving both Articles and Procedures alone.

 

Source: M.I.T. Libraries. Institute Archives and Special Collections. Papers of Charles Kindleberger, 1934-99. Box 22, Folder “Examinations. International Economics, 1959-75”.

Image Source: M.I.T. Yearbook Technique, 1950.

Categories
Exam Questions Johns Hopkins M.I.T. Suggested Reading Syllabus

M.I.T. Readings and exam questions for fiscal and monetary policy. Domar, 1957

Evsey Domar’s first semester at M.I.T. was as a visiting professor according to the teaching records of the economics department. He taught one seminar on Russian Economics (14.292) and a graduate course with the nominal title “Fiscal Policy”. That course had been taught previously by E. Cary Brown (Spring 1954, 1955) and R. A. Musgrave, visiting Professor (Spring 1956).

Inspection of the ten-page course bibliography and the final examination questions along with two note-cards filed with these course materials, it appears that well over half the course was in all likelihood dedicated to fiscal policy topics with monetary policy for stabilization topics accouting for perhaps one-third of the course. Just as the length of the course bibliography (typical for Domar) is daunting, his use of asterisks to designate recommended reading was exceedingly liberal. An examination of the final examination questions leads me to conclude that it should be rather easy to reduce the course reading list (for examination purposes!) to less than two pages.

___________________________

Course Enrollment
(Second term, 1956-57

Instructor

Domar, E. D.

Rank

Prof. (Visit.)

Subj. No.

14.472

Subj. Title

Fiscal Policy

No. Class Hours/Week

3

No. Students

22

Source: M.I.T. Archives, Department of Economics Records 1947-, Box 3, Folder “Teaching Responsibility”.

___________________________

Typed notecards for an introduction to or a review of course.

The traditional arguments regarding the purposes of Monetary Policy:

  1. Stabilization of general prices or of factor earnings—the Wicksell-Davidson controversy. The instrument was the relation between the natural and the market rates.
  2. Stabilization of prices or of employment. Recent literature is full of this.
  3. Stabilization of the general prices or of prices of Federal securities. See Douglas’s and other reports on this recent controversy.
  4. Stabilization of employment or the achievement of growth. Any conflict?
  5. Discretionary methods or automatic provisions? See Simons’ article in Readings in Monetary Theory.
  6. To provide credit and currency, sound and in sufficient quantity.
  7. To protect the international position of the country.
  8. To have special effects, such as:

a. by region
b. by industry
c. by commodity consumed (such as tobacco) or housing
d. on population (by giving exemptions or subsidies for dependents)

  1. Provide revenue [handwritten addition]
  2. Distribution of income [handwritten addition]

*  *  *  *  *  *  *  *  *  *  *  *  *

The following limitations, some real, other imaginary, explain why Fiscal Policy is not as simple as Lerner makes it:

Income distribution
Size of the deficit
Size of the budget
Balance of payments
Special regional and industrial effects
Effects on incentives to work (in inflation)
Automaticity of the system (built-in-flexibility)
Monetary effects (on reserves, deposits)
Long-run effects (on growth and development)

Their presence complicates things and explains all the ingenious articles and tax devices frequently suggested. If not for them, fiscal policy would be very simple indeed: cut taxes or increase taxes, and the same with expenditures.

___________________________

READING LIST
14.472 Fiscal Policy
Spring Term 1956-57

Professor E. D. Domar

PART I—MONETARY POLICY

The purpose of this list is to suggest to the student the sources in which the more important topics in Monetary Policy are discussed from many points of view. His objective should be the understanding of these topics and not the memorization of who said what.

Most of the sources listed here, and particularly the Congressional materials, discuss a number of questions not only in Monetary but in Fiscal Policy as well. Hence it is difficult to classify them.

Items marked with an * are strongly recommended. (I don’t like to use the expression “required” in a graduate reading list.)

  1. Factual Materials on Monetary Problems

Federal Reserve Bulletin.

Treasury Bulletin.

Annual Reports of the Secretary of the Treasury and of the Board of Governors of the Federal Reserve System.

Historical Statistics of the United States, 1789-1945, and the Continuation to 1952.

Congressional Hearings, Reports and other Materials listed below.

  1. Introduction

Hart, Albert Gailord, Money, Debt and Economic Activity, New York 1948.

Hicks*, J. R., “A Suggestion for Simplifying the Theory of Money,” Economica, 1935; reprinted in Readings in Monetary Theory.

Lerner*; Abba P., “Functional Finance and the Federal Debt,” Social Research, 1943, and Readings in Fiscal Policy, p. 468, also Chapter 24 in his Economics of Control, New York, 1944.

Poole, Kenyon E., ed. Fiscal Policies and the American Economy.

Sproul* Allan, “Changing Concepts of Central Banking,” Money, Trade and Economic Growth in Honor of John Henry Williams, New York, 1951.

  1. Monetary Theory and Growth

Gurley*, John G. and Shaw, E. S., “Financial Aspects of Economic Development,” American Economic Review, September 1955, pp. 515-538.

  1. Effectiveness of the Interest Rate

Ebersole*, J. F., “The Influence of Interest Rates,” Harvard Business Review, XVII, i, 1938, pp. 35-39.

Henderson*, R. D., “The Significance of the Rate of Interest,” Oxford Economic Papers, October 1938, I, pp. 1-13.

*Meade, J. E. and Andrews, P. W. S., “Summary of Replies to Questions on Effects of Interest Rates,” Oxford Economic Papers, October 1938, I, pp. 14-31.

Sayers, R. S. “Business Men and the Terms of Borrowing,” Oxford Economic Papers, Feb. 1940, III, pp. 23-31.

Andrews, P. W. S., “A Further Inquiry into the Effects of Rates of Interest,” Oxford Economic Papers, Feb. 1940, III, pp. 32-73.

White*, William H., “Interest Inelasticity of Investment Demand—The Case from Business Attitude Survey Re-Examined,” American Economic Review, September 1956, pp. 565-87.

Lutz, Friedrich A., “The Interest Rate and Investment in a Dynamic Economy,” American Economic Review, Dec. 1945.

  1. General Surveys of Monetary Policy

Federal Reserve Board*, Tenth Annual Report for 1923. See pp. 29-39 particularly.

Chandler*, Lester V., “Federal Reserve Policy and the Federal Debt,” American Economic Review, 1949, and Readings in Monetary Theory, p. 394.

Hardy, Charles O., “Fiscal Operations as Instruments of Economic Stabilization,” American Economic Review, Supplement, 1948, pp. 395-403 and Readings in Monetary Theory, p. 394.

Hart, Albert Gailord, “Monetary Policy for Income Stabilization,” Income Stabilization for a Developing Democracy, ed. by Max F. Millikan, New Haven, 1953.

Williams, John H., “The Implications of Fiscal Policy for Monetary Policy and the Banking System,” AER Proceedings, March 1942; Readings in Fiscal Policy, p. 185.

Smith*, Warren L., “On the Effectiveness of Monetary Policy,” American Economic Review, September 1956, pp. 588-606.

  1. Suggested Objectives and Policies

Hammarskjold, Dag, “The Swedish Discussion on the Aims of Monetary Policy,” reprinted in International Monetary Papers, No. 5, pp. 145-154.

Simons*, Henry C., “Rule versus Authorities in Monetary Policy,” JPE, 1936, and Readings in Monetary Theory, p. 337.

Simons, Henry, “On Debt Policy,” JPE, Dec. 1944, and Readings in Fiscal Policy.

Mints*, Lloyd, W., “Monetary Policy,” Review of Econ. and Stat., 1946 and Readings in Fiscal Policy, p. 344.

Bach*, G. L., “Monetary-Fiscal Policy Reconsidered,” JPE, Oct. 1949, and Readings in Fiscal Policy.

Friedman*, Milton, “A Monetary and Fiscal Framework for Economic Stability,” American Economic Review, 1949, and Readings in Monetary Theory, p. 369.

*United Nations. National and International Measures for Full Employment. Report by a group of experts appointed by the Secretary-General (Lake Success, New York, December 1949).

Viner*, Jacob, “Full Employment at Whatever Cost,” QJE, August 1950, pp. 385-407. Reproduced with omissions in Economic Policy, Readings in Political Economy, edited by William D. Grampp and Emanuel T. Weiler, Homewood, Ill., 1956, pp. 54-65.

Samuelson* Paul A., “Principles and Rules in Modern Fiscal Policy: A New-Classical Reformulation,” Money, Trade and Economic Growth in Honor of John Henry Williams, New York, 1951.

Seltzer* Lawrence H., “Is a Rise in Interest Rates Desirable or Inevitable,” American Economic Review, Dec. 1945; Readings in Fiscal Policy, p. 202.

Roosa, Robert V., “Interest Rates and the Central Bank,” Money, Trade and Economic Growth in Honor of John Henry Williams, 1951.

Roosa*, Robert V., “Integrating Debt Management and Open Market Operations,” American Economic Review, 1952, and Readings in Fiscal Policy, p. 265.

Hansen*, Alvin H., “Monetary Policy,” The Review of Economics and Statistics, May 1955, pp. 110-119.

  1. Commodity Money

Graham, Benjamin, World Commodities and World Currency, New York 1944.

Graham*, Frank D., “Full Employment without Public Debt, Without Taxation, Without Public Works, and without Inflation,” Planning and Paying for Full Employment, edited by Abba P. Lerner and Frank D. Graham, 1946.

  1. Congressional Materials

Joint Committee on the Economic Report. Money, Credit, and Fiscal Policies. Hearings before the Subcommittee on Monetary, Credit and Fiscal Policies of the Joint Committee on the Economic Report, 81st Congress, First Session, September 23, November 16,17,18,22,23 and December 1,2,3,5,7, 1949.

Joint Committee on the Economic Report. Monetary, Credit, and Fiscal Policies. A Collection of Statements Submitted to the Subcommittee on Monetary, Credit and Fiscal Policies by Government Officials, Bankers, Economists, and Others. 1949.

Joint Committee on the Economic Report (The Douglas Subcommittee). A Compendium of Materials on Monetary, Credit, and Fiscal Policies. A Collection of Statements Submitted to the Subcommittee on Monetary, Credit, and Fiscal Policies by Government Officials, Bankers, Economists, and Others. 81st Congress, 2ndSession, Senate Document No. 132, 1950.

Joint Committee on the Economic Report*. Monetary, Credit, and Fiscal Policies. Report of the Subcommittee on Monetary, Credit, and Fiscal Policies of the Joint Committee on the Economic Report. 81st Congress, 2ndSession, Senate Document No. 129, 1950.

Joint Committee on the Economic Report. Monetary Policy and the Management of the Public Debt Hearings before the Subcommittee on General Credit Control and Debt Management of the Joint Committee on the Economic Report, 81st Congress, 2nd Session, March 1952.

Joint Committee on the Economic Report. Monetary Policy and the Management of the Public Debt. Their Role in Achieving Price Stability and High-Level Employment. Replies to questions and other material for the use of the subcommittee on general credit control and debt management. 82nd Congress, 2nd Session, Senate Document No. 123, 1952.

Joint Committee on the Economic Report. Monetary Policy and the Management of the Public Debt Report of the Subcommittee on General Credit Control and Debt Management of the Joint Committee on the Economic Report, 82nd Congress, 2nd Session, 1952.

Joint Committee on the Economic Report. United States Monetary Policy: Recent Thinking and Experience Hearings before the Subcommittee on Economic Stabilization of the Joint Committee on the Economic Report. 83rd Congress, 2nd Session, December 6 and 7, 1954.

Joint Committee on the Economic Report. January 1956 Economic Report of the President. Hearings before the Joint Committee on the Economic Report. 84th Congress, 2nd Session, January 31, February 1,2,3,6,7,8,9,14,15,17 and 28, 1956.

Joint Committee on the Economic Report*. Conflicting Official Views on Monetary Policy; April 1956. Hearings before the Subcommittee on Economic Stabilization of the Joint Committee on Economic Report, 84thCongress, 2nd Session, June 12, 1956.

  1. Readings for Amusement

Outside Readings in Economics, second edition. Selected by Hess, Arleigh P. Jr., Gallman, Robert E., Rice, John P., and Stern, Carl, New York, 1956. The “Dialogue on Money,” by D. H. Robertson; “The Island of Stone Money,” by William H. Furness III; “The Paper Money of Kubla Khan,” by Marco Polo; and “The Edict of Diocletian,” by Humphrey Mitchell, pp. 314-335 are very amusing and instructive.

 

PART II—FISCAL POLICY

See the remarks in Part I.

  1. Factual Materials of General Character

Joint Committee on the Economic Report.* The Federal Revenue System: Facts and Problems, 1956

Treasury Bulletin

Annual Reports of the Secretary of the Treasury and of the Commissioner of Internal Revenue

Statistical Abstract of the United States

Historical Statistics of the United States, 1789-1945 (published by the U. S. Bureau of the Census)

U. S. Treasury Department, Internal Revenue Service, Statistics of Income (an annual publication in two volumes)

U. S. Bureau of the Census, Summary of Governmental Finances (annual series)

The Budget of the U. S. Government

Commerce Clearing House, Inc., Tax Systems

West Publishing Co., Federal Tax Regulations, 1956

Congressional Hearings and Reports, listed in Part I and below

Textbooks on Public Finance and Fiscal Policy

  1. Historical Studies

Ratner, S., American Taxation: Its History as a Social Force in a Democracy, New York, 1942

Fabricant*, S., The Trend of Government Activity in the United States since 1900, New York, 1952, Chapters 1, 6, 7

Studenski, P. and H. E. Kroos, Financial History of the United States, New York, 1952

Musgrave*, R. A. and J. M. Culbertson, “The Growth of Public Expenditures in the United States,” National Tax Journal, June, 1953, pp. 97-115

Paul, R. E., Taxation in the United States, Boston, 1954

  1. Fundamental Assumptions

Hansen*, A. H., “The Stagnation Thesis,” Fiscal Policy and the Business Cycle, New York, 1941, pp. 38-46, and Readings in Fiscal Policy

Schumpeter*, J. A., “Economic Possibilities in the United States,” Capitalism, Socialism, and Democracy, 1947, and Readings in Fiscal Policy

Domar*, E. D., “The Problem of Capital Accumulation,” The American Economic Review, December, 1948

Fellner*, W., “Relative Emphasis in Tax Policy on Encouragement of Consumption or Investment,” Federal Tax Policy for Economic Growth and Stability, Joint Committee on the Economic Report, Washington, D.C., November 9, 1955, p. 210

Hansen*, A. H., “Economic Stability and Growth,” Federal Tax Policy for Economic Growth and Stability, Joint Committee on the Economic Report, Washington, D. C., November 9, 1955, p. 14

Smithies*, A., “Economic Growth as a Policy Objective,” Federal Tax Policy for Economic Growth and Stability, Joint Committee on the Economic Report, Washington, D. C., November 9, 1955, p. 32.

  1. General Objectives and Policies

Keynes* J. M., “An Open Letter,” The New York Times, 1933, and Readings in Fiscal Policy

Lerner*, A. P., “Functional Finance and the Federal Debt,” Social Research, 1943, and Readings in Fiscal Policy. Also Chapter 24 in his Economics of Control, New York, 1944

Hart, A. G., “’Model-Building’ and Fiscal Policy,” American Economic Review, 1945, and Readings in Fiscal Policy

Committee for Economic Development, “Taxes and the Budget: A Program for Prosperity in a Free Economy,” Readings in Fiscal Policy, 1947

Colm* G., “The Government Budget and the Nation’s Economic Budget,” Public Finance, 1948, and Readings

Friedman*, M., “A Monetary and Fiscal Framework for Economic Stability,” American Economic Review, 1948, and Readings

National Planning Association, “Federal Expenditure and Revenue Policy for Economic Stability,” 1949, Readings

Bach*, G. L., “Monetary-Fiscal Policy Reconsidered,” Journal of Political Economy, October, 1949, and Readings

United Nations*, National and International Measures for Full Employment (report by a group of experts appointed by the Secretary-General), Lake Success, New York, December, 1949

Simons*, H. C., Federal Tax Reform, Chicago, 1950

Viner*, J., “Full Employment at Whatever Cost,” The Quarterly Journal of Economics, August, 1950

Samuelson*, P. A., “Principles and Rules in Modern Fiscal Policy; A Neoclassical Reformulation,” Money, Trade, and Economic Growth: in Honor of John H. Williams, New York, 1951

Millikan, M., ed., Income Stabilization for a Developing Democracy, Yale, 1953

Rolph, E.R., The Theory of Fiscal Economics, Berkeley and Los Angeles, 1954

U. S. Congress, Joint Committee on the Economic Report, Federal Tax Policy for Economic Growth and Stability, December, 1955, Hearings

American Economic Association* Readings in Fiscal Policy, Homewood, Illinois, 1955

National Bureau of Economic Research, Policies to Combat Depression, a conference of the Universities-National Bureau Committee for Economic Research, 1956

Council of Economic Advisers*, the latest Annual Report

  1. Institutional Factors

Bailey, S. K., Congress Makes a Law: the Story behind the Employment Act of 1946, New York, 1950

Bailey, S. K. and H. D. Samuel, Congress at Work, New York, 1952

Blough, R., The Federal Taxing Process, New York, 1952

Smithies*, A., The Budgetary Process in the United States, Committee for Economic Development, New York, 1955

  1. Tax Incidence

Musgrave*, R. A., et al, “Distribution of Tax Payments by Income Groups,” National Tax Journal, March, 1951

Little, I. M. D., “Direct versus Indirect Taxes, Economic Journal, September, 1951

Musgrave*, R. A., “On Incidence,” Journal of Political Economy, August, 1953

Bach*, G. L., “The Impact of Moderate Inflation on Income and Assets of Economic Groups,” Federal Tax Policy for Economic Growth and Stability, Joint Committee on the Economic Report, Washington, D. C., November 9, 1955, p. 71

Musgrave*, R.A., “Incidence of the Tax Structure and its Effects on Consumption,” Federal Tax Policy for Economic Growth and Stability, Joint Committee on the Economic Report, Washington, D. C., November 9, 1955, p. 96

  1. Cyclical Aspects

Slichter*, S. H., “The Economics of Public Works,” American Economics Review, 1934, and Readings in Fiscal Policy

Lutz, H. L., “Federal Depression Financing and its Consequences,” Harvard Business Review, 1938, and Readings

Myrdal* G., “Fiscal Policy in the Business Trade,” American Economic Review Supplement, 1939, and Readings

Hagen, E. E., “Timing and Administering Fiscal Policy,” American Economic Review, May, 1948

Committee on Public Issues of the American Economic Association*, “The Problem of Economic Instability,” American Economic Review, 1950, and Readings

Smithies*, A., “The American Economic Association Committee Report on Economic Instability,” American Economic Review, 1951, and Readings

Phillips, A. W., “Stabilization Policy in a Closed Economy,” The Economic Journal, June, 1954, pp. 290-323

Committee for Economic Development*, Problems in Anti-Recession Policy, September 1954

  1. Alternative Budgets for Full Employment

Kaldor*, N., Appendix C in W. H. Beveridge, Full Employment in a Free Society, 1945

Musgrave*, R. A., “Alternative Budget Policies for Full Employment,” American Economic Review, 1945, and Readings

Musgrave*, R. A. and M. H. Miller, “Built-In Flexibility,” American Economic Review, 1948 and Readings

Bishop*, R. L., “Alternative Expansionist Policies,” Income, Employment and Public Policy: Essays in Honor of Alvin H. Hansen, New York, 1948

Stein, H., “Budget Policy to Maintain Stability,” Problems in Anti-Recession Policy, Committee for Economic Development, September, 1954

Hagen*, E. E., “Federal Taxation and Economic Stabilization,” Federal Tax Policy for Economic Growth and Stability, Joint Committee on the Economic Report, November 9, 1955, pp. 58-70

Lusher, D. W., “The Stabilizing Effectiveness of Budget Flexibility,” Policies to Combat Depression, a conference of the Universities-National Bureau Committee for Economic Research, Princeton, 1956

  1. Balanced Budget Multiplier

Wallich*, H. C., “Income-Generating Effects of a Balanced Budget,” Quarterly Journal of Economics, November, 1944

Haavelmo*, T., “Multiplier Effects of a Balanced Budget,” Econometrica, 1945, and Readings

Haberler, G., “Multiplier Effects of a Balanced Budget,” Econometrica, April, 1946

Baumol, W. J. and M. H. Preston, “More on the Multiplier Effects of a Balanced Budget under Full Employment,” American Economic Review, March, 1955

  1. The National Debt

Studenski, P., “The Limits of Possible Debt Burdens—Federal, State, and Local,” American Economic Review, Supplement, 1937

Haley*, R. F., “The Federal Budget: Economic Consequences of Deficit Financing,” American Economic Review, 1941, and Readings

Williams*, H. H., “Deficit Spending,” American Economic Review, February, 1941 and Postwar Monetary Plans and other Essays, 1944

Ratchford, B. U., “The Burden of a Domestic Debt,” American Economic Review, 1942, and Readings

Williams, J. H., “The Implications of Fiscal Policy for Monetary Policy and the Banking System,” Proceedings of the American Economic Association, 1942, and Readings

Domar*, E. D., “The ‘Burden of the Debt’ and the National Income,” American Economic Review, 1944, and Readings

Simons*, H., “On Debt Policy,” Journal of Political Economy, 1944, and Readings

Seltzer, L. H., “Is a Rise in Interest Rates Desirable or Inevitable?” American Economic Review, 1945, and Readings

Wallich, H. C., “Debt Management as an Instrument of Economic Policy,” American Economic Review, June, 1946

Roosa, R. V., “Integrating Debt Management and Open Market Operations,” American Economic Review, 1952, and Readings

Burkhead*, J., “The Balanced Budget,” Quarterly Journal of Economic, 1954, and Readings

  1. Inflation and War Finance

Sprague, O. M. W., “Loans and Taxes in War Finance,” American Economic Review, Proceedings, 1917, and Readings

Keynes*, J. M., How to Pay for the War, London, 1940

Smithies*, A., “The Behavior of Money National Income under Inflationary Conditions,” Quarterly Journal of Economics, 1942, and Readings

Fellner*, W. J., “Postscript on War Inflation: A Lesson from World War II,” American Economic Review, 1947, and Readings

Fetter*, F., “The Economic Reports of the President and the Problem of Inflation,” Quarterly Journal of Economics, 1949, and Readings

Wald, H. P., “Fiscal Policy, Military Preparedness, and Postwar Inflation,” National Tax Journal, 1949, and Readings

Hart, A. G., Defense Without Inflation, New York, 1951

  1. Effect on Incentives: Incentive Taxation

Domar*, E. D., and R. A. Musgrave, “Proportional Income Taxation and Risk Taking,” Quarterly Journal of Economic, May, 1954

Butters, J. K., and J. Lintner, Effect of Federal Taxes on Growing Enterprises, Boston, 1945

Groves*, H. M., Postwar Taxation and Economic Progress, New York, 1946, Chapter 11

Shelton, J. P., and G. Ohlin, “A Swedish Tax Provision for Stabilizing Business Investment,” American Economic Review, June, 1952

Brown*, R. S., “Techniques for Influencing Private Investment,” Income Stabilization in a Developing Democracy, M. Millikan, ed., 1953, pp. 416-432

Domar*, E. D., “The Case for Accelerated Depreciation,” Quarterly Journal of Economics, February, 1953

Butters*, J. K., “Taxation, Incentives, and Financial Capacity,” American Economic Review, Supplement, 1954, and Readings

Brown, E. C., “The New Depreciation Policy under the Income Tax: An Economic Analysis,” National Tax Journal, March, 1955

Goode*, R., “Accelerated Depreciation Allowances as a Stimulus to Investment,” Quarterly Journal of Economics, May, 1955

Break*, G. F., “Effects of Taxation on Work Incentives,” Federal Tax Policy for Economic Growth and Stability, Joint Committee on the Economic Report, Washington, D. C., November 9, 1955, p. 192

Brown*, E. C., “Weaknesses of Accelerated Depreciation as an Investment Stimulus,” Federal Tax Policy for Economic Growth and Stability, Joint Committee on the Economic Report, Washington, D. C., November 9, 1955, p. 495

Butters*, J. K., “Effects of Taxation on the Investment Capacities and Policies of Individuals,” Federal Tax Policy for Economic Growth and Stability, Joint Committee on the Economic Report, Washington, D. C., November 9, 1955, p. 126

Greenewalt*, C. H., “Effect of High Tax Rates on Executive Incentive,” Federal Tax Policy for Economic Growth and Stability, Joint Committee on the Economic Report, Washington, D. C., November 9, 1955, p. 185

Long*, C. D., “Impact of Federal Income Tax on Labor Force Participation,” Federal Tax Policy for Economic Growth and Stability, Joint Committee on the Economic Report, Washington, D. C., November 9, 1955, p. 153

Kaldor*, N., “An Expenditure Tax,” London, 1955

  1. Particular Taxes

Simons, H., Personal Income Taxation, Chicago, 1938

Brown*, E. C., “Analysis of Consumption Taxes in Terms of the Theory of Income Determination,” American Economic Review, March, 1950

Goode*, R., Corporation Income Tax, New York, 1951

Royal Commission on the Taxation of Profits and Income, First Report, February, 1953; Second Report, April, 1954; Final Report, June, 1955

Due, J. F., “Economics of Commodity Taxation and the Present Excise Tax System,” Federal Tax Policy for Economic Growth and Stability, Joint Committee on the Economic Report, Washington, D. C., November 9, 1955, p. 547

Keith*, G., “Economic Impact of the Corporation Income Tax,” Federal Tax Policy for Economic Growth and Stability, Joint Committee on the Economic Report, Washington, D. C., November 9, 1955, p. 658

Goode, R., “The Corporate Income Tax in a Depression,” Policies to Combat Depression, a conference of the Universities-National Bureau Committee for Economic Research, 1956

Merriam, I. C., “Social Security Programs and Economic Stability,” Policies to Combat Depression

Pechman, J. A., “Yield of the Individual Income Tax During A Recession,” Policies to Combat Depression

  1. Inter-Governmental Fiscal Relations

Maxwell*, J. A., “Intergovernmental Fiscal Devices for Economic Stabilization,” Federal Tax Policy for Economic Growth and Stability, Joint Committee on the Economic Report, Washington, D. C., November 9, 1955, p. 807

Heer*, C., “Stabilizing State and Local Finance,” Policies to Combat Depression, 1956

U. S. Treasury Department, Committee on Inter-Governmental Fiscal Relations, Federal, State, and Local Government Fiscal Relations, 78th Congress, 1st Session, Senate Document No. 69, 1943

U. S. Bureau of the Census, Compendium of State Government Finances (an annual series)

Same source, Compendium of City Government Finances (an annual series)

Tax Institute, Federal-State-Local Tax Correlation (A Symposium), December, 1953

The Council of State Governments, Federal Grants-in-Aid, 1949

  1. Growth and Economic Development

Bernstein*, E. M. and I. G. Patel, “Inflation in Relation to Economic Development,” International Monetary Fund, Staff papers, II, 1951-52

United Nations*: Fiscal Division, “Taxation and Economic Development in Asian Countries,” Economic Bulletin for Asia and the Far East, Vol. IV, November, 1953

Gurley, J. A., “Fiscal Policy in a Growing Economy,” Journal of Political Economy, December, 1953

Papers and Proceedings of the Conference on Agricultural Taxation and Economic Development, H. P. Wald, and J. N. Froomkin, eds., Cambridge, Massachusetts, 1954 (Harvard University Law School, International Program in Taxation)

  1. Special Problems

Clark*, C., “Public Finance and Changes in the Value of Money,” The Economic Journal, December, 1945

Clark*, C., “The Danger Point in Taxes,” Harper’s Magazine, December, 1950

Goode*, R., “An Economic Limit on Taxes: Some Recent Discussion,” National Tax Journal, September, 1952

Caplan, B., “A Case Study: The 1948-1949 Recession,” Policies to Combat Depression, 1956

Fox, K. A., “The Contribution of Farm Price Support Programs to General Economic Stability,” Policies to Combat Depression, 1956

Gordon, R. A., “Types of Depressions and Programs to Combat Them,” Policies to Combat Depression

Grebler, L., “Housing Policies to Comat Depression,” Policies to Combat Depression

Johnson, D. G., “Stabilization of International Commodity Prices,” Policies to Combat Depression

Owen, W., “Self-Liquidating Public Works to Combat Depression,” Policies to Combat Depression

Source: Duke University, David M. Rubenstein Rare Book and Manuscript Library. Economists’ Papers Archive. Box 17, Folder “Fiscal and Monetary Policy”.

___________________________

FINAL EXAMINATION
14.472 Fiscal Policy
Monday, May 20, 1957

E. D. Domar

ANSWER ALL QUESTIONS. THE QUALITY OF YOUR REASONING IS THE MOST IMPORTANT PART OF YOUR ANSWERS.

  1. [35%] Compare and contrast monetary and fiscal policies as methods of achieving a steadily expanding economy (without inflation or depression). Include, but don’t limit yourself to, the following points:
      1. The theoretical foundation of each.
      2. Methods used.
      3. Effects on distribution of income and wealth.
      4. Social and political repercussions of each.
      5. The effectiveness and limitations of each.

Do they overlap? Can you work out a synthesis?

  1. [20%] “Government spending tends to be like a drug, in that it takes larger and larger doses to get results, and all the time debt and taxes get higher and higher.”
    Analyze this statement and comment as fully as you can. Compare the effect of government expenditures with that of private.
  2. [15%] “The best cure against inflation is increased production.”
    Analyze this statement and comment on it. Include in your comments the monetary and fiscal implications of this statement.
  3. [15%] What are the so-called “Built-in-Stabilizers?” Discuss fully and indicate how they operate in (a) depression and (b) inflation.
  4. [15%] “The purpose of taxation is never to raise money but to leave less in the hands of the taxpayer.”
    Comment fully and indicate the limitations of this statement. Can you identify the author? (No great penalty if you cannot.)

Source: Duke University, David M. Rubenstein Rare Book and Manuscript Library. Economists’ Papers Archive. Box 16, Folder “Examination. Public Finance and Fiscal Policy”.

Notes on Final Exam:

Question II comes from a review of Stuart Chase, Where’s the Money Coming From? Published in the Monthly Bulletin of the National City Bank of New York that I was fortunate to find inserted into the Congressional Record Volume 93—Part 4 (May 8, 1947, p. 4827);

Question III. Domar liked this question enough to have used it at least twice. See January 23, 1958 Exam at Johns Hopkins; January 26, 1966 at M.I.T.;

Question V. The sentence quoted comes from Abba Lerner’s The Economics of Control, p. 307.

___________________________

Image Source: Evsey D. Domar at the MIT Museum legacy website.

 

Categories
Funny Business Gender M.I.T. Policy Popular Economics

M.I.T. Washington Post op-ed by Samuelson on Sound Debt Policy, 1963

 

Source: Paul A. Samuelson, “We can have sound debt policy” from the Washington Post, included with Extention of remarks of Hon. Jeffery Cohelan of California in the House of Representatives, Friday, May 31, 1963 in Congressional Record: Proceedings and Debates. Volume 109, part 25—Appendix, May 31, 1963, p. A3510

Also found as a mimeographed copy in Harvard University Archives. Papers of Alvin Harvey Hansen, Box 1, Folder “Business Cycles.”

Image Source:  Samuelson Memorial Information Page/Photos from Memorial Service.  Accessed via the Internet Archive Wayback Machine.

Categories
Exam Questions M.I.T.

M.I.T. General Examination in Advanced Economic Theory. Sept 1962 and May 1963

 

 

Edwin Burmeister received an M.A. from Cornell in September 1962 before going on to M.I.T. to complete his Ph.D. in economics in 1965. His papers at the Duke Economists’ Papers Archive include a folder of advanced economic theory general examinations at M.I.T. (May and September 1962; May 1963). The copy of the May 1962 exam has been transcribed and posted earlier. This post adds the remaining two exams to the collection of artifacts. Pro-tip:  Burmeister’s papers includes his solutions to the September 21, 1962 exam, most likely prepared during his preparation for the May 1963 exam.

I should mention that on none of the three exams is “M.I.T.” actually written. However, since Samuelson and Solow’s names are typed on the copy of the Sept 1962 exam and since Burmeister was a M.I.T. graduate student  for certainly the May 1963 examination (and, like many before and after him, cast an eye on previous exam questions), it is pretty obvious where the exam questions must have come from.

________________________

General Examination
Advanced Economic Theory

Professors P. A. Samuelson and R. M. Solow
Friday, September 21, 1962

Do as many problems as you have time.

  1. Derive the demand function, Xi = Di(I, p1, …, pn) ≥ 0 for a consumer with income I and having positive prices and having respectively preferences satisfying the following utility functions:
      1. U = k1 log X1 + … + kn log Xn
      2. U = mX0 +logX1 [Be careful!]
      3. U = a1X1 + a2X2 + … + anXn [where] ai≥0

Extra credit

      1. U = Min (X1/b1, X2/b2, …, Xn/bn)
  1. A firm owning some fixed and non-transferable “capital” has a production function

Q = f(labor, land) = 20L.5T.25

It sells in a competitive market at $Pq. It rents labor in a competitive factor market at $W and rents land at $R.
What are its demand relations for factors, and its supply relation for output? What are its “profits” or “quasi-rents to owned capital.”
It will suffice for you to write down all the relations that define these desired functions and describe how they could be solved. (In other words, you don’t have to do the explicit solving.)

  1. In a Hicksian general equilibrium model all income effects turn out to be negligible. Comment decisively on its

(a) Property of dynamic stability (or possible instability)
(b) Property of imperfect stability (or possible instability)
(c) Property of perfect stability (or possible instability)

  1. Let H(X,y) be a function of non-negative vectors X(of dimension m) and y (of dimension n). Define X*, y* as a saddle point of H if

H(X*,y) ≥ H(X*,y*) ≥H(X,y*)

For all non-negative (X,y).
Prove that X* and y* are optimal vectors for a pair of dual linear programs if and only if they provide a saddle point for the function

H(X,y) = C’X+b’y – y’AX.

Show that a simple Leontief model is capable of producing any positive vector final demands (given enough labor) if and only if (I-A)-1 is non-negative.

  1. Consider the von-Neumann model with 3 activities and 4 commodities and with input matrix

\text{A}=\left[ \begin{matrix} 0 & 1 & 0 \\ 1 & 0 & 0 \\ 0 & 0 & 1 \\ 0 & 1 & 0 \\ \end{matrix} \right] and output matrix \text{B}=\left[ \begin{matrix} 1 & 0 & 0 \\ 0 & 0 & 1 \\ 0 & 2 & 0 \\ 0 & 0 & 1 \\ \end{matrix} \right]

Find the optimal activity and price vectors in the von-Neumann sense, and the associated expansion rate.

________________________

 

General Examination in Advanced Economic Theory: May 1963

Answer any 4 questions.

  1. Suppose all of the N people in a market have identical indifference maps, that are homothetic (i.e., with unitary income elasticities everywhere). Let each jth man have his endowment

\left( \bar{Q}_{1}^{j},\bar{Q}_{2}^{j},\ldots ,\bar{Q}_{r}^{j} \right)

      1. Show that the final equilibrium of exchange is quite independent of the distribution among men of the fixed totals

    \begin{array}{l}\bar{Q}_{1}^{1}+\bar{Q}_{1}^{2}+\ldots +\bar{Q}_{1}^{N}={{A}_{1}}\\...................................\\\bar{Q}_{r}^{1}+\bar{Q}_{r}^{2}+\ldots +\bar{Q}_{r}^{N}={{A}_{r}}\end{array}

    1. Show that the equilibrium prices can be found by treating any man as the single Robinson-Crusoe living under autarky.
    2. What can you, therefore, state about the i) Imperfect, ii) Perfect, and iii) Dynamic stability of the equilibrium?
  1. A Kaldor-Goodwin model defines[sic]
    \text{a}\frac{\text{dK}}{\text{dt}}=\beta \text{Y}-\text{K, }\left( \text{a,b,}\beta \right)>0
    \text{b}\frac{\text{dY}}{\text{dt}}=\frac{\text{dK}}{\text{dt}}-\text{S}\left( \text{Y} \right)
    (i) Explain the meaning of each equation. (ii) Give an equation for its stationary equilibrium solution. (iii) What does its local stability and oscillation depend on? (iv) What shape for the only arbitrary function will give rise to unique-amplitude oscillation?
  2. In Mitopia
    \text{C}+\frac{\text{dK}}{\text{dt}}=\sqrt{\text{KL}}\text{ and L = }{{\text{L}}_{0}}{{\text{e}}^{\text{gt}}}.
    How must K(t) grow if C/L, per capita consumption, is to remain at a maximum constant level? What will then be the interest rate, and the relative share of labor?
  3. A machine with a length of life T costs $f(T). The machine is known with certainty to yield a net income stream of $a per year steadily throughout its lifetime. Find the equation determining the optimal length of life of a machine under each of the following assumptions.
    1. The instantaneous rate of interest in a perfect capital market is r; the length of life is chosen to maximize the present value of net cash flow (including initial cost).
    2. The interest rate r is used to discount net income, and durability is chosen to maximize the capital value of a new machine per dollar of initial cost.
    3. The internal rate of return (i.e. the discount rate that equates capital value and initial cost) is maximized.

Suppose that in cases (a) and (b) the interest rate is such that the capital value of the machine equals its initial cost. Show that all three solutions then coincide. Which is the “right” way to look at the problem?

  1. In a Leontief system with n commodities and one primary factor, labor, let Pi be the money price of commodity i, P0 the money wage, aoi the direct labor input per unit output of commodity i, Xi the output of commodity i, and Ci the final demand for commodity i. Show that the increase in Pj/P0 resulting from a unit increase in a0i equals the increase in Xi needed for a unit increase in Cj.
  2. Consider an individual whose life is divided into two periods, Present and Future. He is endowed with some physical good in each period.
    1. Show how to construct a supply curve relating the amount of saving he will do in the Present as a function of the rate of interest.
    2. Show that in a society of identical individuals with no time preference, the equilibrium rate of interest is zero if corresponding to each individual with endowment X in the Present and Y in the Future, there is another individual with endowment Y in the Present and X in the Future.

 

Source: Duke University. David M. Rubenstein Rare Book & Manuscript Library. Economists’ Papers Archive. Edwin Burmeister papers. Box 23, (unlabeled) Folder.

Categories
M.I.T. Wing Nuts

M.I.T. Wingnut inspiration for Du Pont’s crusade against Paul Samuelson’s textbook, 1947

 

 

What is the natural habitat of wing-nuts and fanatical partisans of zombie economic ideas? While Economics in the Rear-View Mirror specializes in the collection and curation of artifacts bearing on the general academic environment within which economists have been trained in the United States since about 1870, there are moments when a field trip to the lunatic fringe is warranted. It is there where we can observe the margins of the chattering class, working politicians, and wealthy businessmen as they poke their noses into curriculum decisions and professional debates regarding the scope and methods of economics. As the vaudeville comedian Jimmy Durante cracked, “Everyone wants ta get inta da act.”

Executive summary:

Members of the M.I.T. Corporation hostile to Paul Samuelson’s textbook and even the President of M.I.T. appear to have found a kindred spirit in Rose Wilder Lane whose anti-Keynesian review of Lorie Tarshis’ textbook was published in 1947 by the Franco admirer and later John Birch activist Merwin K. Hart.

This post began innocently enough when I selected an exchange of letters concerning the teaching of the principles of economics at M.I.T. in general and the new textbook by Paul Samuelson in particular. The famous controversy involved members of the M.I.T. Corporation, the M.I.T. Administration, and the M.I.T. department of economics and social science and has been most ably presented by Yann Giraud and Roger Backhouse and in the literature they cite.

Yann Giraud. Negotiating the “Middle-of-the-Road” Position: Paul Samuelson, MIT, and the Politics of Textbook Writing, 1945-55. Paper included in MIT and the Transformation of American Economics, Annual Supplement to Volume 46, History of Political Economy edited by E. Roy Weintraub. Durham and London: Duke University Press, 2014, pp. 134-152.

Earlier draft: The Political Economy of Textbook Writing: Paul Samuelson and the Making of the first Ten Editions of Economics (1945-1976). Working Paper 2011-18 of Université de Cergy Pontoise (France).

Giraud’s blog: https://ygiraud.wordpress.com

Also: Roger Backhouse’s Becoming Samuelson (Oxford University Press, 2017), chapter 26.

This post provides a few letters from four of the individuals involved in the Samuelson controversy to provide a taste of that discussion. What caught my eye and what I call the reader’s attention to in this post is the repeated reference to an unnamed critical review of another unabashedly Keynesian textbook, The Elements of Economics by Lorie Tarshis of Stanford University. It is worth noting that Samuelson’s textbook was already receiving incoming fire from members of the M.I.T. Corporation before that review was published in August 1947, so the attack on Tarshis was merely adding water to the Anti-Samuelson mill. The head of the economics department, Ralph Freeman, notes in his defense of Samuelson that the organization that had published the Tarshis review was known to have “a fascist flavor” and was run by a man named Hart who was “involved in some way in a treason charge during the war”. Seeing the words “fascist” and “treason”, I could not resist donning my investigative garb to uncover the back-story of the man Hart, his organization and the anti-Tarshis screed by the author unnamed in the letters. But first I share the sample letters from 1947 in the Samuelson controversy at MIT.

Dramatis Personae

Walter J. Beadle (Vice President, Treasurer and member of the Board of Directors at Du Pont and life member of the M.I.T. Corporation, 1943-88)

Lammot du Pont II (President of Du Pont (1926-40), Chairman of the Board of Directors and former member of the M.I.T. Corporation (1928-33))

President of M.I.T. Karl T. Compton  (b. 14 September, 1887; d. 22 June, 1954)

Head of M.I.T.’s department of economics and social science, Ralph Evans Freeman (b. 23 July 1894; d. 12 May 1967)

Source (DuPont officers): “DuPont Officers Reelected, James New Treasurer Aide” in The Morning News (Wilmington, Delaware) April 22, 1947, p. 12.

Fun Fact:

The great-great grandfather of Lammot Du Pont, the chairman of the Board of Directors at Dupont in 1947, was Pierre Samuel du Pont de Nemours, a disciple of the Physiocrat author of the Tableau Oeconomique, François Quesnay.

The genealogical line from the Physiocrat du Pont de Nemours to the Chairman of the Board of Directors of DuPont in 1947.

Pierre Samuel du Pont de Nemours (b. 14 Dec 1739; d. 7 Aug 1817)

Éleuthère Irénée du Pont de Nemours (b. 24 June 1771; d. 31 Oct 1834)

Alfred V. du Pont (b. 11 Apr 1798; d. 4 Oct 1856)

Lammot du Pont I (b. 13 Apr 1831; d. 29 Mar 1884)

Lammot du Pont II (b. 12 Oct 1880; d. 24 Jul 1952)

 

Image Sources: Pierre Samuel Du Pont de Nemours (Wikipedia Commons); Lamott Du Pont II in Du Pont: The Autobiography of an American Enterprise. New York: Charles Scribner’s Sons, 1952. (Lammot Du Pont, p. 86).

_______________________

Beale to Compton
(original)

Walter J. Beadle
DuPont Building
Wilmington 98, Delaware

September 15, 1947

Dr. Karl T. Compton, President
Massachusetts Institute of Technology
Cambridge, Massachusetts

Dear Dr. Compton:

When you were on vacation, Mr. C. E. Spencer, Jr. sent me a copy of the Economic Council Review of Books for August 1947. Since this seemed to point up better than anything I have read the general problem in connection with teaching of economics in this country, I sent it to Jim Killian in advance of our luncheon meeting and he in turn passed it on to Professor Freeman.

On the chance that you have not seen this review, I attach a copy of it which has just come to me from Mr. Lammot du Pont. I enclose also Mr. du Pont’s letter of transmittal dated September 12th which I am sure will be of interest to you. As I told Jim at our Boston meeting, I acquainted Mr. du Pont with the developments in connection with the teaching of economics at M.I.T. because I know of his very sincere interest in the Institute as a life member of the Corporation.[sic, not listed as a Life Member At MIT’s website]

I hope that your vacation proved to be a very enjoyable and refreshing one.

With kind regards, I am

Sincerely,
[signed] Walter
Walter J. Beadle

WJB:k
enc.

Source: MIT Archives. Office of the President Box 192, Folder 9 “Samuelson, Paul, 1942-1947”.

_______________________

Lammot Du Pont to Beadle
(copy)

LAMMOT DU PONT
Du Pont Building
Wilmington 98, Delaware

September 12, 1947

Mr. Walter J. Beadle;
B u i l d i n g.

Dear Walter:

Your file is returned herewith, and there is also enclosed a leaflet of the National Economic Council, giving a review of college textbook, “The Elements of Economics,” by Lorie Tarshis. You can get an idea of the nature of the review by reading the few paragraphs on the first page, which I have marked.

I take it that this textbook is an aggravated example of what the M.I.T. professor [Paul Samuelson] has done in a milder way. You will note on page 7 a list of the colleges which have adopted this textbook, and I am pleased to note that M.I.T. is not among them. Will you use your judgment as to sending this copy of the review to Dr. Compton as an illustration of what can happen?

Recently, I was talking with an Economist, who is a professor at a well-known university in the east. I have entire confidence in this Economist’s truthfulness and accuracy, but maybe I did not understand him exactly right. The gist of what he told me was as follows:

At this university there are 11 professors in the Department of Economics. Of these, 7 are Leftist. Four, including himself, are what I would call “sound.” There are two vacancies among the 11 professorships, and it is indicated that they will be filled only with men who meet with the approval of the present 9 incumbents. This is called “a democratic process.” With the odds 7 to 2, it is a foregone conclusion that another Leftist will be added.

In addition to the above, my friend tells me that he has been advised by a man acting as Assistant to the President of the University, with respect to faculty appointments, that he, my friend, had better withdraw from the University, or look for a position elsewhere. My friend informs me that he does not intend to withdraw, and does not think they can oust him. He believes that it is his duty to remain at the University and do what he can to expound to students sound economics. The University is among those listed on page 7 of this leaflet.

I am not urging that you send this review to Dr. Compton, or that you send him this letter, but if you care to do so, you have my permission, for I don’t think I have violated any confidence in what I have written.

Yours sincerely,
(s) Lammot du Pont

LduP/MD

Source: MIT Archives. Office of the President Box 192, Folder 9 “Samuelson, Paul, 1942-1947”.

_______________________

Compton to Beadle
(copy)

September 18, 1947

Mr. Walter J. Beadle
du Pont Building
Wilmington 98, D.C. [sic]

Dear Walter:

Thanks ever so much for sending me the copy of the Economic Council Review of Books for August, which discusses the book by Professor Tarshis of Stanford University.

My brother Wilson showed me a copy of this while we were together at our family camp, and I had made a memorandum to send for a copy for my own use. It seems to me to be an exceedingly effective statement.

Incidentally, have you noticed the comment among the book reviews in the September issue of Fortune with reference to another book by one of Samuelson’s students [Lawrence Klein]?

I am just getting squared away after return from vacation and the process is somewhat delayed because I got mixed up in a fire and am still somewhat bandaged up,–nothing permanently serious, however.

With best regards,

Very sincerely yours
[unsigned]
President

KTC/L

Source: MIT Archives. Office of the President Box 192, Folder 9 “Samuelson, Paul, 1942-1947”.

_______________________

Compton to Freeman
(copy)

December 15, 1947

Personal

Professor R.E. Freeman
Dept. of Econ. and Soc. Sci.

Dear Ralph:

Apropos of the discussions which we had some weeks ago about Professor Samuelson and the textbook on economics, I have accidentally run into several interesting discussions recently concerning the Keynesian theories of economics on the part of several groups of top economists. From these I gained the impression that Keynes’ theories were brilliant and stimulating but inclined to be based more on a logic derived from a limited set of postulates than on actual test from all the factors involved. The comment was made that Lord Keynes himself was sufficiently flexible to modify his views when the facts indicated to him that this was necessary, but that many of Keynes’ disciples have been so wedded to the beautiful logic that they have had a tendency to base their faith on this logic rather than on an objective evaluation of factors by which the conclusions might be tested.

The work of the American Economic Council [sic], (I am not sure that I have the name just right), was described as especially valuable and effective because of its objective search for facts, as opposed to argument on theory.

At a meeting with Harold Moulton some weeks ago I asked his opinion of Samuelson and he replied that Samuelson is a very brilliant young man but that he is a “dogmatist”. In this connection Moulton dug out the enclosed reprint which he thought might be helpful to us in our evaluation of economic research methods. I thought you might be interested in this, though you have perhaps already read it. Please return it at your convenience,

Very sincerely yours,
[unsigned]
President.

KTC/L

Source: MIT Archives. Office of the President Box 93, Folder 7 “Freeman, R.E. 1940-1944”.

_______________________

Freeman to Compton
(original)

Personal

Massachusetts Institute of Technology
Department of
Economics and Social Science

Cambridge, Mass.
17 December 1947

Dr. Karl T. Compton
Room 3-208
M.I.T.

Dear Dr. Compton:

Many thanks for your comments regarding Keynes, Samuelson et al. I was interested in Moulton’s brochure which I am returning herewith.

A good deal of misunderstanding has arisen because of a failure to distinguish between Keynesianism as a conceptual apparatus and Keynesianism as a policy. It is the former which has been adapted by the younger economists of this country such as Paul Samuelson—and many of the older ones as well. I use the word adapted, because some of the ideas of Keynes have been rejected. On the policy level two Keynesians may arrive at quite different conclusions.

The charge that such thinkers base their faith on logic rather than on facts, is to my mind unjustified. The classical economists built up their whole system on the assumption of full employment. The modern approach is not only to question this assumption but also to try to understand why our economy so often fails to provide full employment.

It has been a common belief in the past that because the rate of saving was assumed to vary with the interest rate, there could be no under or over savings—that changes in the interest rate would provide the necessary correction. A study of the facts indicates that this position was erroneous. Much of “modern economics” is concerned with the implications of under-saving and over-saving.

I have taken the liberty of enclosing a recent bulletin of the United Business Service for which I write the first page every week. This brief article designed for popular consumption entitled “How Inflation Could Be Halted” illustrates the use of the savings concept in analyzing current problems. Incidentally, Moulton in the latter part of the pamphlet you sent me indicates that he has incorporated into his thinking the Keynesian approach to the saving process.

It is significant, I believe, that the new approach to economic problems has developed as our knowledge of the facts of the economic process has become more extensive. Today we know vastly more about what is going on in economic society than we did a half or even a quarter of a century ago. The young men who have been and are now the main fact gatherers are in overwhelming numbers using the Keynesian concepts as tools of analysis.

The “American Economic Council” [sic] to which you refer in your letter is I believe an organization with a Fascist flavor which is of course opposed to the “new economics.” If I have identified the organization correctly, it is a front for a man named Hart who was involved in some way in a treason charge during the war. It recently issued a review of a book by Tarshish [sic]—a review which was grossly unfair to the writer.

I am not sure what Moulton means by referring to Paul Samuelson as “dogmatic.” Paul certainly is capable of supporting his views with factual data and reasoned arguments. Moulton’s effort to defend a recent Brookings publication—“A National Labor Policy”—against the criticism of Wayne Morse was not an effort which would inspire confidence in Moulton’s own objectivity.

I don’t know whether Bob Caldwell passed to you the information that Paul will be presented with the John Bates Clark Medal at the coming meetings of the American Economic Association in Chicago. This medal is being presented for the first time by the Association to the living economist under 40 “who has made the most distinguished contribution to the main body of economic thought and knowledge.” The name of the recipient of the award will not be published until December 28.

Probably you will agree with me that we don’t need to worry too much about what the economists of the country think about Paul Samuelson.

Sincerely yours,
[signed] Ralph
Ralph E. Freeman

 

Source: MIT Archives. Office of the President Box 93, Folder 7 “Freeman, R.E. 1940-1944”.

_______________________

Compton to Freeman
(copy)

December 19, 1947

Professor Ralph E. Freeman
Department of Economics and Social Science
M. I. T.

Dear Ralph:

Thanks ever so much for your letter and the enclosed copy of United Business Service.

One way or another I seem to be getting some elements of an education in economics, long deferred. At least no one can criticize my own education in this field on the ground that it has not brought contact with plenty of divergent points of view.

I was glad to have your distinction between conceptual apparatus and policy in reference to the influence of Lord Keynes.

I am delighted to know that Paul Samuelson is to receive the John Bates Clark Medal. That, coming from the American Economic Association, is certainly an honor and should be a reassurance to some of our “worriers”.

With many thanks,

Sincerely yours,
[unsigned]
President

KTC/h

 

Source: MIT Archives. Office of the President Box 93, Folder 7 “Freeman, R.E. 1940-1944”.

_______________________

Back to the Chase

Thanks to my reading of Giraud and Backhouse, it didn’t take much effort to establish the identity of the unnamed reviewer of Tarshis, none other than the libertarian diva, Ms. Rose Wilder Lane (b. 5 December 1886; d. 30 October 1968). Economics in the Rear-View Mirror has posted the story of Rose Wilder Lane’s 1946 report for the Foundation of Economic Education on Milton Friedman and George Stigler’s famous pamphlet on rent-control, Roofs or Ceilings. Lane was certain that Messrs. Friedman and Stigler were communists in deep disguise…really. Interested readers can find out more about her together with the complete text to the third printing of her 1947 review of Tarshis in the rich paper with its document-filled appendix by Levy, Peart and Albert (2012).

David M. Levy, Sandra J. Peart and Margaret Albert. Economic Liberals as Quasi-Public Intellectuals: The Democratic Dimension in Marianne Johnson (ed.) Documents on Government and the Economy Vol. 30-B (2012) of Research in the History of Economic Thought and Methodology, pp. 1-116.

Especially the transcription of the Rose Wilder Lane review of the textbook The Elements of Economics by Lorie Tarshis published in Economic Council Review of Books, Vol. IV, No. 8, August 1947), pp. 49-64.

More about Merwin Kimball Hart can be found at:

Sandra J. Peart and David M. Levy. F. A. Hayek and the “Individualists”, Chapter 2 in F. A. Hayek and the Modern Economy: Economic Organization and Activity, eds. Sandra J. Peart and David M. Levy (Palgrave Macmillan, 2013), especially pp. 30-37.

_______________________

But wait, there’s more

For those wanting to learn even more about the publisher of the National Economic Council’s Review of Books, Mr. Merwin Kimball Hart (b. 25 June 1881; d. 30 November 1962), U.S. government files are available at archive.com that were obtained through Ernie Lazar’s FOIA applications. There you will find around six hundred pages of F.B.I. investigative reports, letters, and newspaper clippings regarding the Merwin Hart case that are easily consulted on line.

The tidbit that I find that ties this post together is the clear evidence that Lammot Du Pont was a financial supporter of Hart’s National Economic Council precisely at the time that he and the Du Pont vice-president and lifetime member of the M.I.T. corporation were on a crusade against Paul Samuelson’s textbook.  “A rose by any other name would smell as sweet.”

 Links to the Merwin Kimball Hart files

Hart, Merwin K.—NYC 100-21056 (243 pages)

14 page New York City F.B.I. investigative report November 17, 1942
6 page Albany F.B.I. report Jan 22, 1943 on Utica background of Merwin K. Hart

Hart, Merwin K.—HQ 100-128996, Misc. Serials (278 pages)

[note:it is necessary to view the file in single-page mode, when in double page mode only the odd numbered pages are displayed.]

Hart, Merwin Kimball, HQ 100-128996, 139-142 (58 pages)

Hart, Merwin K.—Army Intel Report (48 pages)

A selection from these FOIA files now follows:

_______________________

A Memorandum for the Director of the F.B.I. (February 8, 1940) prepared by E. A. Tamm

The FBI report refers to a woman informant working within the New York State Economic Council.

“From what can be gathered from the informant the Council was apparently originally engaged in a fight against Communism. It then became involved in the fight to support the Franco rebellion in Spain, and has now passed into not only opposition to the present Federal administration but has gone further and become actually opposed to the existing form of government in this country. The inner circle of the NYSEC in one way or another is now considering setting up an independent union movement to combat the CIO and other so-called radical unions, and to set up what would amount to company-controlled unions.

The informant advises [deletion of 2/3 line] Hart, John Eoghan Kelly, Jane Anderson, and various Catholic priests, she is convinced of the existence of a plot, presumably centering around the Council and directed by Catholic church leaders to reestablish the Holy Roman Empire with certain nations so aligned as to make it possible for the Catholic church to control the balance of power through its control of the government of Spain.” Page 3 of memo

[…]

“Hart is general manager of the Cream of Wheat Corporation, and his home is understood to be at Utica, New York…The informant expresses her belief that Hart is a sincere, fiery patriot who honestly believes the country is in serious danger from a “red menace.” However, she stated he is being used by certain Fordham University clerics who decide on certain action in conferences with John Eoghan Kelly, Allen Zoll and similar persons, and then prevail on Hart to make such contacts, presumably Protestant, as will facilitate the promotion of the action desired.

“Hart has written a book entitled ‘America—Look at Spain’, and from purported copies of correspondence exhibited by the informant it would appear that this book was partly edited by the Catholic clergy in so far as that portion of it which treats the Catholic church is concerned. Hart has visited Spain, Germany and Italy and has made an intensive study of conditions in these places. He has communicated with the Bureau in the past relative to cooperating on matters pertaining to the national defense. By letter dated April 10, 1939 he wrote the Bureau requesting a copy of the report on the German-American Bund investigation, and was advised that same was not available, and his letter was referred to the Department.

It is impossible to fully set out all the connections that Hart may possibly have, but it is probably safe to say from those he is known to have that he is connected at least with every group of any prominence in the United States whose aims are anti-administration or anti-Communist.” Page 5 of memo.

[…]

DuPonts of Wilminton, Delaware:

            The informant advises that these persons were at one time strong financial supporters of the NYSEC but have not contributed recently.” Page 21 of memo.

 

Source: Memorandum for the Director of the F.B.I. (19 February 1940) in the Ernie Lazar FOIA Collection at archive.org. Federal Bureau of Investigation, N.Y.C. Hart, Merwin K.—HQ 100-128996, Misc. Serials.

_______________________

From an investigative report dated July 2, 1942

“…On January 27,1940 Confidential Informant [deleted] was interviewed by Assistant Director E. J. Connelley regarding any information informant might have concerning MERVIN K. HART. Informant informed [ca. 2 lines deleted] whose offices are located in Room 417, 17 East 442 Street, New York City. Informant advised that she met subject HART through [deleted] who was an acquaintance of HART as a customer of the bank [deleted] started to work for HART [line deleted] HART advised informant that he had just returned from Spain where he was in touch with the Nationalist Leader and believed that they were saving the world form Communism. He wanted to write a book to show that the same thing might occur here in the United States.

She advised that HART had published a book entitled AMERICA LOOKS AT SPAIN which was published by Kennedy and Company. HART advised informant that in this book he wanted to show that Communism was overthrowing the world and that something must be done about it in this country. In connection with the luncheon held for MARTIN DIES, which was mentioned previously, [one line deleted] this luncheon for Dies was given by the New York State Economic Council at the Bellmore Hotel, New York City. Informant advised that JAMES WHEELER-HILL, Second in Command of the German-American Bund, was there along with [deleted] The luncheon was open to the public. She stated that the presence of [deleted] and JAMES WHEELER-HILL did not mean that they were connected with the Economic Council as tickets were on sale to the public; however, informant said that the people actively working for HART considered [deleted] and WHEELER-HILL as martyrs fighting for a cause.

Informant said [deleted] he formed the American Union for Nationalist Spain and, in that connection, was constantly in touch with various religious leaders. Informant, continuing, said that the Council is financed through subscriptions and donations made by the Texas Company and by Lamont [sic] Dupont. According to informant, HART’s most intimate associate is Captain JOHN T. TRAVER, the head of the American Coalition of Patriotic Societies.

 

Source: Ernie Lazar FOIA Collection at archive.org. Federal Bureau of Investigation. NY File No. 100-21056. Report date: 2 July 1942.

_______________________

From November 17, 1942 FBI Internal Security Case Report
Merwin K. Hart

…Confidential Informant [deleted] stated that she first met HART during the winter of 1938-1939 at a party at the home of [deleted] of the famous [deleted] of China. HART at that time had just returned from Spain. [Deleted] had just returned from Munich and was disgusted with the Chamberlain appeasement policy. She thereafter disliked HART’S theories from the start. For quite some time HART continued to send her a copy of his Economic Letter, which she said she tore up and refused to pay any attention to it. According to [deleted] HART has constantly criticized the ROOSEVELT administration; is violently anti-Communistic; has said that HITLER has done some good things for Germany; that the German American Bund is a harmless organization; and that the Franco Policy is satisfactory. She said further, however, that since December 7, 1941 HART has been openly advocating unity withi9nAmerica. He confines his criticism now only to Government spending and then only to expenditures which are not for the war effort. However, she believes he is still a Fascist in his theories of Government but is smart enough to hold his tongue now. She said that a while ago he was so anti-Communistic he was literally seeing “a Communist under every chair.” She believes he might still be regarded as dangerous in that his constant criticisms creates a disturbing element. She does not believe that he is subsidized by foreign funds. She said further that HART had told her in the past of attending some Bund meetings simply to find out what went on in the meetings. [Deleted] was of the opinion that HART’S theories are too extreme, and that HART has been and in her opinion still is against labor agitation…”

 

Source: Ernie Lazar FOIA Collection at archive.org. Federal Bureau of Investigation. NY File No. 100-21056. Report date: 17 November 1942.

_______________________

Memo for J. Edgar Hoover Jan 26, 1944.

 

Item in summary table of correspondence with Merwin K. Hart:

From Lammot du Pont to M.K.H. 1/2/42. Encloses check for $4,000. “Subscription to the work of the organization for 1942”

Source: Ernie Lazar FOIA Collection at archive.org. Federal Bureau of Investigation. NY File No. 100-21056. Memo to Hoover (26 January, 1944).

_______________________

Newspaper clipping, syndicated columnist Marquis W. Childs

Marquis W. Childs. The State of the Nation.
[FBI time stamp: Jan 15, 1948]

Washington.

The self-appointed thought police are on the loose again, their attack this time is directed against a textbook on economics used in many of the leading universities of the country.

The attack began with the National Economic Council, whose head, Merwin K. Hart, has been one of the principal American supporters of Spain’s dictator, Franco. It took the form of a so-called review of the book—“The Elements of Economics” by Prof. Lorie Tarshis of Stanford University.

The review twists the meaning of the book to try to show that its author supports the government spending theories of the late Lord Keynes. Therefore, the review concludes, the book must be subversive and un-American.

Wide circulation of this review through the mails was only the first step. In Arkansas, an American Legion post and something called the Arkansas Free Enterprise association have taken the next step. They have demanded an investigation of the textbook, used in economics classes at the University of Arkansas.

President Lewis W. Jones of the university replied that he thought the sanest procedure would be to submit the book to an impartial group capable of judging it, such as the American Economics [sic] association. He added that he saw nothing subversive in the text, which he considered a thoroughly objective study of the economic system.

Here is a pattern of behavior that endangers fundamental American freedoms of speech and thought. The concept of thought police, whether amateur or professional, is repugnant to free Americans.

The American legion recently held here in Washington a counter-subversive seminar. Seventy-five representatives from Legion posts around the country attended the three-day session. They heard lectures by some so-called experts on Communism. It is interesting incidentally, that among these experts are several men who were once Communists. Having at one time embraced a totalitarian faith, they now make a profession of denouncing it.

Seven State Legion organizations have held or will hold such seminars, taking their cue from the National organization. Both Georgia and Indiana have just had two-day sessions on subversion.

If one is to judge from the speech made by Georgia’s Rep. James C. Davis at the meeting in Atlanta, it was given over entirely in the subversion of communism. They might well have devoted part of their time to such home-grown subversion as the Ku Klux Klan. It is a fairly safe guess that there are more Klansmen than Communists in Georgia.

Training Legionnaires to “spot and counter subversive activities, as National Commander O’Neil put it, is a hazardous business. The FBI gives its agents months of instructions in such matters, and they are told to avoid possible infringement of fundamental rights of speech and thought. Yet here we have amateurs turned loose after two days to do sleuthing on their own.

An example of what this can mean occurred in California at about the time the Legion was holding its counter-subversive seminar in Washington. Twenty-five men wearing Legion hats bearing the insignia of Glendale, Cal., Post No. 127 invaded the meeting of a Democratic club and demanded that it break up immediately.

A slight error had been made. The club was duly chartered by the County Democratic Central committee. In the midst of the indignation and the corresponding embarrassment that followed, State Legion Commander Harry L. Foster condemned the act.

“The rights of free speech and assembly,” he said, and it might be a good idea to frame these words in every Legion hall, “are part of our cherished Bill of Rights and we of the Legion should be the first to insist on these rights. Should there is an unlawful meeting, it should be reported to the duly constituted civil authorities for their action.

“Thought police on the Japanese model are an insult to American integrity. That is especially true when zealous guardians of pure thought seek to protect the young. If young men and women in college who have grown up under the advantages of the American system cannot use judgment for themselves, then the system has failed. The generation that fought the recent war does not need to be sheltered by meddling zealots. They are a lot more clear-eyed and clear–headed than most of their elders.

 

Source: Ernie Lazar FOIA Collection at archive.org. Federal Bureau of Investigation. N.Y.C. File No. 100-21056 page 179.  Copy from the FOIA file is partially illegible and the newspaper was not identified. A less edited version of the article was published in The Eau Claire Leader (Wisconsin), Sunday, January 18, 1948, p. 12.

_______________________

J. Edgar Hoover’s Memo
March 29, 1948

100-128996-94

Date: March 29, 1948

To: [deleted]

BY SPECIAL MESSENGER

Attention: Reading Center

From: John Edgar Hoover, Director, Federal Bureau of Investigation

Subject: MERWIN KIMBALL HART, wa,
Mervin Kimball Hart
National Economic Council, Inc.
INTERNAL SECURITY-X

Reference is made to your communication of March 17, 1948 your [deleted] where you informed that [deleted] was en route to New York City at the invitation of the National Economic Council.

[paragraph deletion]

Biographical information, the accuracy of which is unknown, reflects that Merwin Kimball Hart was born on June 21, 1881, at Utica, New York. He graduated from Harvard in 1904, receiving an A. B. degree. In 1906 he was elected for a two year term to the General Assembly of the State of New York. Hart, by this time, was married to Catherine Margaret Crouse of Utica. He was admitted to the New York State Bar in 1911 and became a member of the law firm Hart and Senior. In 1914 Hart and several prominent businessmen in Utica organized the Utica Mutual Insurance Company. A few years later when the United States entered the war, Hart, although possessing defective eyesight, enlisted in the Army and when released in 1918 he had attained the rank of Captain in a non-combatant unit. After the war Hart devoted several years attempting to place the Hart and Crouse Company in Utica on a sound financial basis. This firm, which manufactures furnaces and heating equipment, was founded by Hart’s father and Hart’s wife’s father. The firm is presently owned by other persons. Following this, Hart became active in numerous movements to reduce expenditures in the State government of New York. Subsequently in about 1932 he organized the New York State Economic Council, now known as the National Economic Council, Inc., with offices in New York City and Utica, New York. His annual salary from the inception thereof was reported to be $10,000. The organization was originally financed by manufacturing and financial concerns located in the State of New York.

Hart was described by an old acquaintance as having come from one of the old established families in Utica, was a brilliant and well educated man was thoroughly patriotic and loyal, and taken part in numerous business enterprises, and was one time a member of one of the leading law firms in Utica. In this latter connection this informant stated it was not known whether Hart went to law school or that he ever appeared in court as a lawyer.

Another source stated Hart was very influential and respected in his own community, but had few intimate friends. He said Hart was known as the type who knew “everybody that counted” and acted in a formal and aloof manner. His personal unpopularity in Utica was attributed in part to the fact that he was too outspoken, tactless, bull-headed, and possessing a peculiar type of personality.

Hart was described as believing in the capitalistic system and particularly opposed to Communism and the New Deal Administration. It was said that the citizens of Utica generally considered him sincere and 100% American in spite of his unfavorable publicity. Some people, it was claimed who did not know him, might think him to be opposed to the country’s war aims at that time.

Information of a current nature regarding the National Economic Council, Inc., is not known. From various sources in the past it was described as being an organization of about 17,000 members drawn from throughout the State of New York. Its headquarters were said to have been located at 17 East 42nd Street, New York City, with a branch office at Utica. A folder distributed by the Council in 1940 described as the Council’s purposes: 1. To curb Government spending; 2. To reduce oppressive tactics; 3. To oppose subversive groups; 4. To oppose stifling restrictions of private enterprises, and 5. To promote true recovery. The officers of the Council as listed in the folder are as follows: President—Merwin K. Hart; Treasurer—George D. Graves; Vice-President— [name missing] Chase National Bank, New York City; Chairman of the Finance Committee—William Fellows Morgan, New York City; Vice-Presidents—Elon Hooker—President Hooker Electrochemical Company, New York City; Thomas M. Peters, New York City; Alexander D. Falck, Chairman, Corning Glass Works, Elmira, New York.

A confidential source advised that early in 1940 the headquarters of the Council seemed to be a meeting place for groups of people who were apparently interested in setting up a totalitarian form of government. This organization was also said to furnish material to Reverend Charles Coughlin for his use. Starting in late 1939 it was reported that the Council devoted about 90% of its efforts to the distribution of propaganda on behalf of the Spanish Republican Government.

The answer to question “d.” is not known to this Bureau. Accordingly, appropriate inquiry is being instituted in an effort to ascertain the desired information. Upon receipt of the results of this inquiry I shall promptly advise you.

Source: Ernie Lazar FOIA Collection at archive.org. Federal Bureau of Investigation, N.Y.C. Hart, Merwin K.—HQ 100-128996, Misc. Serials.  pp. 187-189.

 

_______________________

WASHINGTON CITY NEWS SERVICE
[teletype]
File Time Stamp: August 14, 1950

MERWIN K. HART, PRESIDENT OF THE NATIONAL ECONOMIC COUNCIL, INC., SAID TODAY THE WORD “DEMOCRACY” IS CLOSELY ASSOCIATED WITH COMMUNISM AND SHOULD BE DISCARDED.

HE TOLD THE SPECIAL HOUSE COMMITTEE INVESTIGATING LOBBYING THAT THE U.S. IS A REPUBLIC AND THAT “IT IS TIME FOR US TO RETURN TO THAT CONCEPTION.”

THE TERM “DEMOCRACY” GAINED ITS CURRENT STATUS AFTER IT WAS USED BY GEORGEI DIMITROV AT A MEETING OF THE COMMUNIST INTERNATIONAL IN MOSCOW IN 1935, HART SAID.

REPEATING WHAT HE SAID IN A SPEECH TO THE UNION LEAGUE CLUB OF NEW YORK IN 1940, HART TOLD THE COMMITTEE:

“I WONDER SOMETIMES IF ONE OF THE CAUSES OF OUR TROUBLE TODAY DOES NOT ARISE FROM THE FACT THAT WE HAVE BEEN OVER-DRILLED INTO BELIEVING WE ARE A DEMOCRACY, THIS, TOO, MAY BE ONE OF THE LATEST ‘INSIDIOUS WILES OF FOREIGN INFLUENCE…IT IS TIME TO BRUSH ASIDE THIS WORD WITH ITS ‘CONNOTATIONS.’”

HART WAS CALLED BEFORE THE LOBBY COMMITTEE BECAUSE OF THE EFFORTS MADE BY HIS ORGANIZATION TO INFLUENCE LEGISLATION IN WHICH IT IS INTERESTED. THE COUNCIL IS CLASSIFIED BY BOTH DEMOCRATIC AND REPUBLICAN CONGRESSMEN AS RIGHT-WING.

IN ONE OF SEVERAL PREVIOUS STATEMENTS MADE BY HART, WHICH WERE PUT INTO THE COMMITTEE RECORD, HE SAID THERE IS AN “EXTREMELY ACTIVE GROUP” ATTEMPTING TO CONVERT THE UNITED STATES FROM A REPUBLIC TO A DEMOCRACY—“THAT IS, FROM A REPRESENTATIVE FORM OF GOVERNMENT INTO A MOBOCRACY, GOVERNED EVENTUALLY BY A DICTATOR.”

ALSO PUT INTO THE COMMITTEE RECORD WERE NUMEROUS EXCHANGES OF LETTERS IN WHICH CONTRIBUTIONS AND GIFTS TO THE NATIONAL ECONOMIC COUNCIL WERE DISCUSSED.

THE LETTERS SHOWED THAT TWO OF THE ACTIVE CONTRIBUTORS TO THE COUNCIL ARE LAMMOT DU PONT AND IRENEE DU PONT, BOTH OF WILMINGTON, DEL. THE RECORDS SHOWED THAT IRENEE DU PONT GAVE THE COUNCIL $11,000 IN 1948 TO PAY FOR SUBSCRIPTIONS TO PAMPHLETS THAT WERE SENT TO COLLEGES, CHURCHES AND LIBRARIES.

HART SAID IN ONE LETTER TO FORMER U.S. SEN. JOESPH R. GRUNDY, OF BRISTOL, PA., THAT THE COUNCIL’S LEGAL STAFF HAD FOUND A METHOD OF HELPING ITS CONTRIBUTORS SAVE ON THEIR INCOME TAX PAYMENTS.

“MAY I SAY THAT WHILE UNDER A RULING OF THE TREASURY DEPARTMENT OUR NON-NEW DEAL NATIONAL ECONOMIC COUNCIL IS NOT ABLE TO OFFER THE DEDUCTIBILITY PRIVILEGE TO ITS CONTRIBUOTRS, YET WE ARE ABLE TO GET SUBSTANTIAL BENEFIT FROM THE FACT THAT A CONTRIBUTION MADE TO US OF MONEY TO PURCHASE SUBSCRIPTIONS AT $10 EACH TO OUR COUNCIL PUBLICATIONS TO GO TO EDUCATIONAL AND RELIGIOUS CORPORATIONS IS DEDUCTIBLE UNDER THE INCOME TAX LAW,” HART WROTE GRUNDY.

HART’S LETTER SAID THAT FROM TIME TO TIME GRUNDY HAD SHOWN INTEREST IN THE NATIONAL ECONOMIC COUNCIL. THE FORMER PENNSYLVANIA SENATOR WAS INVITED TO MAKE A “FAIRLY SUBSTANTIAL” CONTRIBUTION TO THE WORK OF THE COUNCIL.

THERE WAS NO EVIDENCE PRODUCED AT THE COMMITTEE HEARING TO SHOW WHAT GRUNDY’S REPSONSE WAS.

6/21—N122P

Source: Ernie Lazar FOIA Collection at archive.org. Federal Bureau of Investigation. HART, Merwin Kimball, HQ 100-128996, 139-142.

ADD 1 LOBBYING (122P)

THE HOUSE LOBBY INVESTIGATING COMMITTEE DISCLOSED THAT CONFIDENTIAL INFORMATION ABOUT ONE OF ITS SECRET MEETINGS HAD LEAKED OUT TO A LOBBY WHICH IT IS INVESTIGATING.

HARRY S. BARGER, WASHINGTON REPRESENTATIVE OF THE NATIONAL ECONOMIC COUNCIL, DECLINED TO TELL THE COMMITTEE HOW HE GOT INSIDE INFORMATION ABOUT THE COMMITTEE’S JANUARY 17 MEETING.

IN A MEMO FROM BARGER TO ERWIN K. HART, NEC PRESDIENT, BARGER SAID “A FRIEND OF MINE” SAW A REPORT OF THE MEETING. BARGER DECLINED TO NAME THE FRIEND AND ASKED THE COMMITTEE FOR A RULING ON WHETHER HE WOULD BE COMPELLED TO ANSWER.

CHAIRMAN BUCHANAN SAID THE PROBLEM WOULD BE TAKEN UP IN CLOSED SESSION.

IN BARGER’S MEMO, INTRODUCED AS EVIDENCE, HE WROTE HART THAT THE COMMITTEE HAD FOUND THAT $90,000 HAD BEEN CONTRIBUTED TO NEC “FROM THE DUPONTS,” AND THAT THE COMMITTEE THOUGHT NEC WAS “SOMEWHAT SUBVERSIVE IN CHARACTER.”

BARGER WROTE THAT “THE CIO AND KINDRED SPIRITS” WERE RUNNING THE COMMITTEE AND “THAT THE SETUP SHOULD BE VERY CAREFULLY EXPOSED IF AND WHEN REPRESENTATIVES OF THE COUNCIL ARE CALLED BEFORE THE BUCHANAN COMMITTEE X X X.”

REVELATION OF THE MEMO BROUGHT SHARP COMMENTS FROM COMMITTEE MEMBERS, ESPECIALLY REP. CLYDE DOYLE, D., CAL., WHO DECLARED “I EMPHATICALLY RESENT” THE CHARGE THAT THE COMMITTEE IS UNDER DOMINATION OF ANY ONE.

BARGER SAID THE INFORMATION ABOUT THE COMMITTEE’S SECRET MEETING “WAS GIVEN TO ME IN CONFIDENCE” AND COULD HAVE COME FROM ANY ONE OF “THREE OR FOUR FRIENDS.”

“I DON’T THINK I SHOULD BE CALLED UPON TO NAME MY SOURCES ANY MORE THAN A NEWSPAPER MAN SHOULD BE,” HE SAID.

6/21—WM611P

Source: Ernie Lazar FOIA Collection at archive.org. Federal Bureau of Investigation. HART, Merwin Kimball, HQ 100-128996, 139-142.

_______________________

And no counterrevolution would be complete without the guns
Reported June 1950 in the Washington Post

It was brought out, however, that Hart warned subscribers in his economic council letter in January, 1948, to arm themselves with pistols and rifles to resist the Communist threat.

“We have one concrete suggestion to make to every citizen who is impressed by the potential danger,” he wrote. “Let him possess himself of one or more guns making sure they are in good working condition and that other members of his family know how to use them.”

After the letter was read, Hart explained it had been written after a trip to Europe. He said it seemed to him that laws against the possession of firearms discriminate against law-abiding citizens because Communists and others ignore them.

Washington Post clipping “circa 6/_/50, p. 5.

Source: Ernie Lazar FOIA Collection at archive.org. Federal Bureau of Investigation. HART, Merwin Kimball, HQ 100-128996, 139-142.

Categories
Cal Tech Columbia Fields M.I.T. Michigan Princeton Stanford

Columbia. Memo advocating the establishment of an Industrial Relations Section. Wolman, 1944

 

 

The following brief memo written by Leo Wolman was commissioned in 1943 by an informal committee to provide a case for establishing an Industrial Relations Institute at Columbia. Besides identifying the existing centers of industrial relations research and teaching in the U.S. and Canada, Wolman also points to the key role played by “C. J. Hicks, the dean of American industrial relations men, adviser to the Rockefellers on policies and problems in this field and, until his retirement some 15 years ago, the director of labor relations for the Standard Oil Co. of New Jersey.”

_____________________

Leo Wolman, Biographical Note

1890, Feb. 24. Born, Baltimore, Md.
1914. Ph.D. in political economy, Johns Hopkins University, Baltimore, Md.
1916. Published The Boycott in American Trade Unions. Baltimore: Johns Hopkins Press
1918. Appointed head of section on production statistics, War Industries Board
1919. Attached to American peace mission, Paris, France
1919-1928. Member, faculty, New School for Social Research, New York, N.Y.
1920-1931. Director of research, Amalgamated Clothing Workers Union
1920-1934. Editor, Journal of American Statistics Association
circa 1925. Became freelance researcher for the National Bureau of Economic Research, formally joining the staff in 1931 and later becoming director-at-large for research. NBER publications by Leo Wolman.
1931-1958. Professor of economics, Columbia University, New York, N.Y.
1933. Appointed to staff of National Recovery Administration
1936. Published Ebb and Flow in American Trade Unionism. New York: National Bureau of Economic Research
1961, Oct. 2. Died, New York, N.Y.

Source: Library of Congress. Leo Wolman Papers. Biographical Note.

_____________________

COPY TO DR. FACKENTHAL

October 23, 1944

Dean George B. Pegram,
201 Low Memorial Library.

Dear Dean Pegram:

I enclose a copy of a statement prepared by Professor Wolman on “Industrial Relations Sections or Departments in American Universities”. This was prepared in compliance with the recommendation made by the informal committee that met last year to consider the possibility of our setting up an Industrial Relations Institute at Columbia. I have had some two dozen copies of this statement mimeographed. These will be available for distribution if you plan to call another meeting to explore this matter further.

Faithfully yours,

_____________________

Industrial Relations Sections or Departments

During the past 15 years, a number of American universities, and one Canadian, have organized sections or departments of industrial relations. The earliest of these was the Industrial Relations Section of Princeton University. Since 1930, similar sections have been established at the University of Michigan, Stanford, California Institute of Technology, Massachusetts Institute of Technology, and Queens University, Canada. These sections are integral parts of the graduate departments of the several institutions. The moving spirit in initiating and finding financial resources for the sections, already established, was C. J. Hicks, the dean of American industrial relations men, adviser to the Rockefellers on policies and problems in this field and, until his retirement some 15 years ago, the director of labor relations for the Standard Oil Co. of New Jersey.

The purposes of this departure were several—to keep members of the faculty and students abreast of the very rapid developments in this important area of private and public policy, to make available to employers, managers, labor, and public officials comparative data as to practices, rules, procedures and policies, to enable students desiring to specialize in labor, labor relations and related subjects to observe and study the practical workings of industrial relations, to push forward the boundaries of knowledge through research, and to establish a closer relation between the scientific activities of universities and the problems of industry, labor, government, and the public. In carrying out these purposes, the various sections have built up libraries of current materials, have published studies dealing with current developments, such as the reemployment of veterans, or of historical importance, such as labor banking in the United States, have trained graduate students, and have held conferences, annual as a rule, for persons working in labor relations.

Depending on their location, age, and industrial environment, the sections now in operation have emphasized different practices. California Technology, operating in a region where large-scale industry is relatively new and personnel men are scarce, has devoted much of its time and resources to bringing to bear the knowledge and experience of other parts of the country on the problems and needs of Southern California. The Massachusetts Institute, operating in an area concerned with unemployment and industrial contraction, has concentrated on research in wages, labor mobility, unemployment, and the like. But all of the sections study, teach, and write about the large issues of private and public policy.

The funds for these enterprises come largely from business, usually in the form of annual contributions pledged for periods of 3 or 5 years. Occasionally a specific piece of research is financed by one of the Foundations but this source of funds has not been counted on for current expenses. Contributions by labor unions have been only a small fraction of total income, though they generally participate in the conferences, and make use of available materials.

There can be little question that the establishment of an industrial relations section at Columbia (associated with the faculties of Political Science and Business) would confer many benefits upon the University. It would make available to students in this field facilities, publications, and contacts with labor and industry which they now lack. It would open up for graduate students new opportunities for employment. It would make available to the university facilities and funds for research. It would create for interested numbers of the faculty, working in the related areas of labor economics, theory, public law, sociology, and labor law, the occasions for using the materials, experience, and problems of industry, labor, and government, not now available to them. It would enable the University to enlarge the range of its public service by serving some of the needs of the enormous and variegated industry, located in this city and the surrounding industrial area of New Jersey, Connecticut, and New York State.

The funds for such an undertaking are probably available in industry. At any rate the other universities had no difficulty raising money. What is needed at Columbia is endorsement of the idea by the faculty, administration, and trustees and the appointment of a small committee instructed to make the plans, raise the funds and find the man capable of directing a section of industrial relations at Columbia.

Leo Wolman

Source: Columbia University Libraries, Manuscript Collections. Columbiana. Department of Economics Collection. Faculty. Box 2, Folder “Department of Economics—Faculty. Beginning Jan. 1, 1944”.

Image Source: Detail from a faculty group picture (early 1930’s). Columbia University Libraries, Manuscript Collections. Columbiana. Department of Economics Collection. Box 9, Folder “Photos”.

Categories
Economics Programs Faculty Regulations M.I.T.

M.I.T. “Industrial Economics” Ph.D. name changed to “Economics”, Economics S.M. recognized as exit ramp, 1965

 

Somewhat surprising is the late date (1965!) of the name-change for the economics Ph.D. at M.I.T. from “Industrial Economics” to “Economics”. Also interesting in the transcribed memorandum below is the request to lower the math and science prerequisites for the economics S.M. to that of the Ph.D. in order to facilitate the graceful, early exit of graduate students unlikely to complete the Ph.D. 

____________________

MASSACHUSETTS INSTITUTE OF TECHNOLOGY
Department of Economics and Social Science

MEMORANDUM

February 5, 1965

To: Committee on Graduate School Policy
From: Robert L. Bishop, Head, Department of Economics and Social Science

For some time now, there has been a strong sentiment in our Department that our graduate degree programs should be changed and supplemented. The changes that we should now like to propose officially will require action by the Faculty and the Corporation, because they involve changes in degree titles, in one instance a change in prerequisites and content of the degree, and in another instance the addition of a new degree. On the other hand, the changes are not really of a radical nature and will not involve any additional staff or any augmenting of the numbers of our graduate students.

At present we have programs for a Ph.D. in Industrial Economics, a Ph.D. in Political Science, and an S.M. in Economics and Engineering or in Economics and Science. Our proposals are: (1) to change the title of the Ph.D. in Industrial Economics to a Ph.D. in Economics; (2) to substitute for the present S.M. degrees a single S.M. in Economics, with admission requirements the same as for the Ph.D. in Economics; and (3) to add an S.M. in Political Science, having an analogous relationship to the existing Ph.D. in Political Science.

Dropping the adjective “Industrial” from the title of our Economics Ph.D. is merely a belated recognition of the considerable broadening of that program that has taken place since it was first established in the years just prior to World War II. At that time, the designation of Industrial Economics appropriately reflected the limited kind of study that was then visualized. Since then, however, our program has expanded in its scope and diversity so that the original designation has become a decided anachronism for the majority of our Ph.D. recipients.

Even in the beginning, as now, the admission requirements for our Economics Ph.D. have differed from those in most Departments, in that they did not include the amount of mathematics and science taken by M.I.T. undergraduates. Instead, only one full year of college mathematics and one full year of college work in science have been required. These requirements reflect, of course, a desire to make our program accessible to most Economics majors in liberal arts colleges. The requirements for our present S.M. degrees, by contrast, constitute essentially the subjects taken by an undergraduate in the Economics option of Course XIV. It is those admission requirements that we propose to change, so that a candidate for the Ph.D. might alternatively be a candidate for an S.M.

Professional training for a career in Economics is such that the Ph.D. has really become the essential degree for anyone who aspires to the fullest professional status. Nor is it our intention to admit candidates solely for the S.M., except in very special circumstances. Over the years, however, we have felt the desirability of being free to award and S.M. in Economics to some students. These include some foreign students, often connected with research programs at the Center for International Studies, who can profit significantly from graduate study at M.I.T. but who are unable to stay long enough for the full Ph.D. program. In all frankness, too, it must be confessed that we have sometimes wished that we were free to divert a Ph.D. candidate toward the lesser degree because of inadequacies of performance after enrollment at the institute. Naturally, the student whose performance is acutely disappointing should not be given any favorable consideration. In many cases, however, performance is not up to the high standard that I think we have maintained for the Ph.D., but still high enough to merit continuance for an S.M.

The reasons supporting a new S.M. degree in Political Science are exactly the same. The only difference here is that there is no S.M. of any kind available in Political Science.

These changes involving S.M. degrees are also in line with some changes that we are simultaneously proposing to the Committee on Curricula with respect to our undergraduate degrees in Course XIV. It is being proposed that these degrees be redesignated more simply as in Economics (Course XIV-A) or in Political Science (Course XIV-B). A copy of these proposals is attached.

Provided that both the undergraduate and graduate program changes are approved, we shall then adopt the same distinction between Course XIV-A and Course XIV-B at the graduate level as at the undergraduate. This will achieve the important administrative reform of distinguishing, as is not now the case, the Economics and Political Science graduate students.

As to the details of the revised graduate degrees, I enclose alternative catalogue copy that would replace the descriptions on pages 142-144 in the present catalogue.

To the extent that the Committee on Graduate School Policy may wish some further discussion of these changes, my colleagues and I will be very pleased to provide it.

RLB:e

 

Source: MIT Institute Archives. Department of Economics records. Box 1, Folder “Comm. On Grad. School Policy”.

Categories
Exam Questions M.I.T. Suggested Reading Syllabus

M.I.T. First core graduate macroeconomics. Syllabus, readings, exams. Domar and Harris, 1967-68

 

 Four out of the five times that the first term of the macroeconomics sequence at M.I.T. (Theory of Income and Employment) was taught in the second half of the 1960’s, it was taught by Evsey Domar . Earlier posts with materials for Domar’s course include the reading list and final exam for 1960-61, reading list and exams for 1965-66 , the exams for 1968-69, and the course evaluations for 1967/68-1969/70.

Responsible for the course section in 1967-68 was the assistant professor John Rees Harris (b. 1934, d. 2018, 1967 Northwestern Ph.D. in economics) [copy of his c.v. archived 14 February 2019]. Here is link to a video lunchtime talk by Harris at the Boston University conference “Development that Works” (March 11, 2011). The picture is a screen-capture from the video.

______________________

M.I.T.
THE THEORY OF INCOME AND EMPLOYMENT
14.451
1967-68
[first session]

I. ADMINISTRATIVE QUESTIONS

    1. Course number, my and Harris’s name, our office numbers, office hours Tu 2:30-3:30.
    2. Sitting chart. No compulsory attendance.
    3. Reading list. First part only. Required and recommended or optional. Responsible for all required reading, but not for the details. I don’t know them myself. Lectures are the skeleton of the course. Reserve in Dewey. Inform me if some books are absent.
    4. The National Income problem. It is due….
    5. Midterm exam in November. Final exam.
    6. Other administrative problems?

II. THE PURPOSE AND NATURE OF THE COURSE

To fill in the gaps and bring everyone to a common denominator, without pulling anyone down. Hence, some will find it a bit boring. Attendance is not compulsory.

It is an introductory course. Almost everything will be discussed in other courses, except National Income, Index of Industrial Production, etc. Growth and fluctuations; monetary economics, consumption function, investment decisions, etc.

III. COMMENTS ON MACROECONOMICS

At the beginning, was a very hot subject—the most interesting part of economics. Two reasons: (1) it was new: (2) the greatest deficiency was in the macro area. Emphasis in those days was on full employment, not growth. Growth came in after the second world war.

The close connection between macro economics and governmental policies.

Three [sic] aspects:

(1) understanding of macro problems by economists

(2) persuading the public—easy in England, very difficult here.

(3) Forecasts of the future—improvement

(4) The effectiveness of methods—also part of forecasts.

On the whole macro-policy has been very successful, sometimes by design, sometimes by luck. The tax reduction of 1964 was the first one for fiscal policy specifically. Less fear of a deficit—witness the present situation. But the tax rise is still a test.

Next step—economic growth. First models—macro type with one kind of goods, and investment with capital coefficients. Still being used, but they don’t get us far.

Growth is to a considerable extent a micro-problem, or at least a mixture of the two. Much more difficult for the government to legislate. How does one improve efficiency? Evaluation of investment projects, of economic effects of education, etc.

Some exaggeration—but the traditional macro theory suffers from its own success.

 

PART I NATIONAL INCOME AND RELATED ITEMS

First—to state the objectives, such as welfare (whose?), capacity to produce (what?), national prestige, evaluation of policies, curiosity about growth, etc.

How to bring order out of the chaos? Which goods and services, which transactions are to be recorded?

Define the purpose of economic activity:

(1) Welfare of all people (or citizens) of a given area

(2) Welfare of some people only (slaves or relatives excluded). Weights?

(3) Welfare of animals? The old lady and her cat?

The definition of welfare may lead to a definition of activities to be included.

Special activities: warfare (Sparta), capital formation, police protection, etc.

Market vs. non-market goods. Imputed items.

 

Source: Duke University. David M. Rubenstein Rare Book and Manuscript Library. Economists’ Papers Archive. Evsey D. Domar Papers. Box 17, Folder “Macroeconomics. Theory of National Income and Employment”.

_________________________

THEORY OF INCOME AND EMPLOYMENT
14.451
Fall Term 1967-68

E.D. Domar
J.R. Harris

READING LIST

The purpose of this list is to suggest to the student the sources in which the more important topics of the course are discussed from several points of view. His objectives should be the understanding of these topics and not the memorization of opinions and details.

The “optional” reading has been included for those students who wish to pursue some of the subjects in greater detail. Some of the items on the optional list may be more effective in their exposition, at least for some individuals, than those on the required list.

There exists a good (if a bit obsolete) textbook on macroeconomics—Gardner Ackley, Macroeconomic Theory (The Macmillan Company, New York, 1961). Its knowledge is necessary but not sufficient for passing the course. While several copies are on reserve at Dewey, the acquisition of private copies is recommended.

Students may also find it convenient to acquire the following books: Readings in Macroeconomics edited by M.G. Mueller (which contains a number of relevant articles) and possible the three National Income volumes published by the U.S. Department of Commerce and listed in Section I.

I. NATIONAL INCOME AND RELATED ITEMS
(September 19 – October 12)

REQUIRED

Ackley, Chapters 1-4.

Kuznets, S., National Income and Its Composition, Vol. I (New York, 1941), Chap. 1.

National Income 1954 Edition, A Supplement to the Survey of Current Business, U.S. Department of Commerce (Washington, D.C., 1954), pp. 27-60, 153-58.

U.S. Income and Output, A Supplement to the Survey of Current Business, U.S. Department of Commerce (Washington, D. C., 1958), pp. 50-105.

The National Income and Product Accounts of the United States, 1929-1965. U.S. Department of Commerce (Washington, D.C., 1966). Browse through the statistics tables of the three volumes to find out what is available where.

Bergson, A. The Real National Income of Soviet Russia since 1928, Ch. 3 on “Methods and Procedures”, (Cambridge, Mass., 1961).

Griliches, Z. “Notes on the Measurement of Price and Quality Changes”, in Models of Income Determination, Studies in Income and Wealth, Vol. 28 by the Conference on Research in Income and Wealth, National Bureau of Economic Research, 1964, pp. 381-418.

Leontief, W. W., “Output, Employment, Consumption and Investment,” Quarterly Journal of Economics, Vol. 58 (February, 1944), pp. 290-314.

Leontief, Studies in the Structure of the American Economy (New York, 1953), pp. 27-35.

Dorfman, R., “The Nature and Significance of Input-Output,” Review of Economics and Statistics, Vol. 36 (May, 1954), pp. 121-33.

Domar, E. D., “On the Measurement of Technological Change,” The Economic Journal, Vol. 71 (December, 1961), pp. 709-29. [Read only pp. 709-14, 726-29.]

Board of Governors of the Federal Reserve System, Industrial Production 1959 Revision (Washington, 1960), pp. iii-41. [Look for the method, not for statistical details.]

Domar, E. D., “An Index-Number Tournament,” The Quarterly Journal of Economics, Vol. LXXXI (May, 1967), pp. 169-88.

Sigel, S. J., “A Comparison of the Structures of Three Social Accounting Systems,” National Bureau of Economic Research, Input-Output Analysis: An Appraisal, The Conference on Research in Income and Wealth, Studies in Income and Wealth, Vol. 18 (Princeton, 1955), pp. 253-89.

 

OPTIONAL READINGS:

Jaszi, G., “The Statistical Foundations of the GNP,” Review of Economics and Statistics, Vol. 38 (May, 1956), pp. 205-14.

Lewis, Wilfred, Jr., “The Federal Sector in National Income Models,” and comments by Hickman and Pechman, in Conference on Research in Income and Wealth, Models of Income Determination (Princeton, 1964), Vol. 28, pp. 233-78.

Bailey, M. J., National Income and the Price Level (New York, 1962), pp. 269-300.

Kuznets, S., National Income and Its Composition (New York, 1941).

Ruggles, R. and N., National Income Accounts and Income Analysis (New York, 1956).

Ruggles, “The U.S. National Accounts,” American Economic Review, Vol. 49, (March, 1959), pp. 85-95.

National Bureau of Economic Research, The National Economic Accounts of the United States, Review, Appraisal and Recommendations, General Series 64, (Washington, 1958).

Organization for European Economic Cooperation, A Standardised System of National Accounts, (Paris, 1952).

Gilbert, M. and I. B. Kravis, An International Comparison of National Products and the Purchasing Power of Currencies, A Study of the United States, the United Kingdom, France, Germany and Italy, Organization for European Economic Cooperation (Paris, 1954).

Gilbert, M., Comparative National Products and Price Levels, A Study of Western Europe and the United States, Organization of European Economic Cooperation, (Paris, 1958).

United Nations, Yearbook of National Accounts Statistics, the latest issue.

United Nations, National Income Statistics, the latest issue.

United Nations, World Economic Survey and other Economic Surveys.

Studenski, The Income of Nations. Theory, Measurement, and Analysis: Past and Present (New York, 1958). [A wealth of information, particularly of historical character.]

Nove, A., “The United States National Income A La Russe,” Economica, Vol. 23, 1956.

Bergson, A. The Real National Income of Soviet Russia Since 1928 (Cambridge, Massachusetts, 1961). (The rest of the book).

Kravis, I. B., “Relative Income Shares in Fact and Theory,” American Economic Review, Vol. 49 (December, 1959), pp. 917-49.

Samuelson, P. A., “Evaluation of Real National Income,” Oxford Economic Papers (New Series), 1950, pp. 1-29.

Samuelson, “The Evaluation of ‘Social Income’: Capital Formation and Wealth,” in F. A. Lutz and D. C. Hague, editors, The Theory of Capital (London, 1961).

Leontief, W. W., The Structure of American Economy (New York, 1941).

Leontief, Studies in the Structure of the American Economy (New York, 1953).

Taskier, C. E., Input-Output Bibliography 1955-1960, United Nations (New York, 1961).

Evans, W. D., and M. Hoffenberg, “The Interindustry Relations Study for 1947,” Review of Economics and Statistics, Vol. 34, (May, 1952), pp. 97-142.

Stewart, I. G., “The Practical Uses of Input-Output Analysis,” Scottish Journal of Political Economy, Vol. 5, (February, 1958).

Dosser, D. and A. T. Peacock, “Input-Output Analysis in an Under-Developed Country: A Case Study,” Review of Economic Studies, Vol. 25 (October, 1957).

Input-Output Analysis: An Appraisal, Studies in Income and Wealth by the Conference on research in Income and Wealth, Vol. 18 (Princeton, 1955).

Solow, R. M. “Technical Change and the Aggregate Production Function,” Review of Economics and Statistics, Vol. 39 (August, 1957), pp. 312-20.

Abramovitz, M., “Resources and Output in the United States Since 1870,” American Economic Review, Papers and Proceedings, Vol. 46 (May, 1956), pp. 5-23, reprinted as National Bureau of Economic Research, Occasional Paper 52 (New York, 1956).

Kendrick, J. W., Productivity Trends in the United States (Princeton, 1961).

Denison, E. F., Sources of Economic Growth in the United States and the Alternatives Before Us (New York, 1962).

Abramovitz, M., “Economic Growth in the United States,” American Economic Review, Vol. 52 (September, 1962), pp. 762-82. [This is a review of Denison’s Book.]

Moorsteen, R. H., “On Measuring Productive Potential and Relative Efficiency,” Quarterly Journal of Economics, Vol. 75 (August, 1961), pp. 451-67.

Fabricant, S., The Output of Manufacturing Industries, 1899-1937 (New York, 1940), particularly Chapter 1.

United Nations, Statistical Office, Index Numbers of Industrial Production, St/Stat/ Ser/ F1 (New York, 1950).

Board of Governors of the Federal Reserve System, Flow of Funds in the United States 1939-53 (Washington, D. C., 1955).

Powelson, J. P., National Income and Flow-Of-Funds Analysis (New York, 1960).

Measuring the Nation’s Wealth, National Bureau of Economic Research, Studies in Income and Wealth, Vol. 29 (Washington, D. C., 1964).

 

READING LIST—SECOND INSTALLMENT
II. GENERAL AGGREGATIVE SYSTEMS—FIRST APPROXIMATION
(October 17 – October 31).

REQUIRED:

Ackley, Parts II and III.

Keynes, J. M., The General Theory of Employment, Interest and Money (London and New York, 1936). [Omit the appendixes to Chapters 6 and 19.]

Note: Neither book is arranged in the order of this reading list. Hence these two assignments apply to other sections of it as well.

Wells, P., “Keynes’ Aggregate Supply Function: A Suggested Interpretation,” The Economic Journal, Vol. 70 (September, 1960), pp. 536-42.

Johnson, H. G. and the discussants, “The General Theory After Twenty-five Years,” American Economic Review Papers and Proceedings, Vol. 60 (May, 1961), pp. 1-25.

Klein, L. R., “The Empirical Foundations of Keynesian Economics,” in K. K. Kurihara, ed., Post Keynesian Economics(New Brunswick, N. J., 1954), pp. 277-319.

 

OPTIONAL READINGS:

Lekachman, Robert, Keynes’ General Theory: Reports of Three Decades, (New York and London, 1964).

Patinkin, D., Money, Interest, and Prices, Second Edition, (New York, 1965).

American Economic Association, Readings in Business Cycle Theory (Philadelphia, 1944), Essays 5, 7, 8.

American Economic Association, Readings in the Theory of Income Distribution (Philadelphia, 1946), Essay 24.

Metzler, “Three Lags in the Circular Flow of Income,” in Income, Employment and Public Policy, Essays in Honor of Alvin H. Hansen (New York, 1948), pp. 11-32.

Harris, S. E., The New Economics (New York, 1947), Essays 8-19, 31-33, 38-46.

Lerner, A. P., Economics of Control (New York, 1944), Chapters 21-23, 25.K

Kurihara, K. K., Post Keynesian Economics (New Brunswick, N. J., 1954).

Klein, L. R., The Keynesian Revolution, (New York, 1947), Chapters 3-5.

Ellis, H. S., A Survey of Contemporary Economics, Vol. 1, (Philadelphia, 1948), Chapter 2.

Burns, A. F., “Economic Research and the Keynesian Thinking of Our Times,” in his The Frontiers of Economic Knowledge, (Princeton, 1954), or in the Twenty-Sixth Annual Report of the National Bureau of Economic Research, Inc.(New York, 1946). See also the discussion by Hansen and Burns in the Review of Economic Statistics (November, 1947).

Dillard, D., “The Influence of Keynesian Economics on Contemporary Thought,” American Economic Review, Papers and Proceedings, 1957.

Hutt, W. H., Keynesianism: Retrospect and Prospect (Chicago, 1963).

Friedman, Milton, and G. S. Becker, “A Statistical Illusion on Judging Keynesian Models,” Journal of Political Economy, Vol. 55 (February, 1957), pp. 64-75.

 

III. PRICE FLEXIBILITY AND EMPLOYMENT
(November 2-9)

REQUIRED:

Patinkin, D., Money, Interest, and Prices, Second ed., (New York, 1965), Chapters 9-11.

Pigou, A. C., “The Classical Stationary State,” Economic Journal (December, 1943).

Power, J. H., “Price Expectations, Money Illusion and the Real Balance Effect,” Journal of Political Economy, Vol. 67 (April, 1959).

Mayer, T., “The Empirical Significance of the Real Balance Effect,” Quarterly Journal of Economics, Vol. 73 (May, 1959).

 

OPTIONAL READINGS:

Readings in Monetary Theory, Essay 13.

Schelling, T. C., “The Dynamics of Price Flexibility,” American Economic Review (September, 1949).

Lange, O., Price Flexibility and Employment (Bloomington, Indiana, 1944). [Get the main idea and omit the details.]

Friedman, M., “Lange on Price Flexibility and Employment,” American Economic Review (September, 1946).

Patinkin, D., Money, Interest, and Prices (Evanston, Illinois, 1956).

Hicks, J. R., “A Rehabilitation of ‘Classical Economics’,” Economic Journal, Vol. 47, (June, 1957).

 

IV. The Theory of Interest and the Demand for Money

Required:

Keynes, General Theory, Chapters 13-17.

Hansen, A., Monetary Theory and Fiscal Policy, Chapters 3,4.

Hicks, J. R., Value and Capital, Chapters 11, 12.

Friedman, M., “The Quantity Theory of Money—A Restatement,” Studies in the Quantity Theory of Money.

Patinkin, D., Money, Interest and Prices, 2nd ed., Chapters VIII, XV.

Tobin, J., “Liquidity Preference as Behavior Towards Risk,” The Review of Economic Studies, February 1958, pp. 65-86.

 

Optional:

American Economic Association, Readings in the Theory of Income Distribution (Philadelphia, 1946), Essays 22, 23, 26.

American Economic Association, Readings in Monetary Theory, (New York, 1951), Essays 6, 11, 15.

Friedman, M. and A. J. Schwartz, A Monetary History of the United States 1867-1960 (Princeton, 1963).

Gurley, J. G., and E. S. Shaw, “Financial Aspects of Economic Development,” AER, vol. 65, September 1955, pp. 515-38.

Gurley, J. G., and E. S. Shaw, Money in a Theory of Finance (Washington, 1960).

Hart, A. G., and P. B. Kenen, Money, Debt and Economic Activity, Third Ed., (Englewood Cliffs, N. J., 1961).

Lydall, H., “Income, Assets, and the Demand for Money,” Review of Economics and Statistics, vol. 40, February 1958, pp. 1-14.

Lutz, F. A., “The Interest Rate and Investment in a Dynamic Economy,” AER, December 1945).

Matthews, R. C. O., “Liquidity Preference and the Multiplier,” Economica, vol. 28, February 1961, pp. 37-52.

Patinkin, D., “Liquidity Preference and Loanable Funds: Stock and Flow Analysis,” Economica, Vol. 25, November 1958.

Review of Economics and Statistics Supplement, vol. 45, February 1963, on “The State of Monetary Economics.”

Wright, A. L., “The Rate of Interest in a Dynamic Model,” Quarterly Journal of Economics, vol. 72, August 1958, pp. 327-50.

 

Reading List—Third Installment
V. Consumption and Saving

Required:

Clower, R.W., “The Keynesian Counterrevolution: A Theoretical Appraisal,” in Hahn and Brechling (eds.), The Theory of Interest Rates (Macmillan, 1965).

Davidson, P., “A Keynesian View of Patinkin’s Theory of Employment,” E.J., September 1967.

Leijonhufvud, A., “Keynes and the Keynesians: A Suggested Interpretation,” AER, May 1967.

Ackley, Chapters 10, 11, 12.

Keynes, General Theory, Chapters 8, 9, 10.

Hagen, E.,”The Consumption Function: A Review Article,” Review of Economics and Statistics, XXXVII, Feb. 1955, pp. 48-54.

Duesenberry, J. S., Income, Saving, and the Theory of Consumer Behavior, Chapters 3, 4.

Friedman, M., A Theory of the Consumption Function, Chapters 1, 2, 3, 9.

Ando, A. and Modigliani, F., “The ‘Life Cycle’ Hypothesis of Saving,” AER, March 1963, pp. 55-85; March 1964, pp. 111—113.

Farrell, M. J., “The New Theories of the Consumption Function,” E.J., vol. 69, December, 1959, pp. 678-96.

Lintner, J., “The Determinants of Corporate Saving,” Savings in the Modern Economy (W. Heller, ed.), pp. 230-55.

Lintner, J. and discussants, “Distribution of Income of Corporations Among Dividends, Retained Earnings, and Taxes,” AER, vol. 46, May 1956, pp. 97-118.

Friend, I., and Kravis, I.B., “Entrepreneurial Income, Saving and Investment,” AER, vol. 47, June 1957, pp. 269-301.

Lubell, H., “Effects of Redistribution of Income on Consumers’ Expenditures,” AER, vol. 37, March 1947, pp. 157-170.

________, “A Correction,” AER, vol. 37, December 1947, p. 930.

Domar, E. D., Essays in the Theory of Economic Growth (New York, 1957), pp. 154-67, 195-201.

Bronfenbrenner, Yomana and Lee, “A Study in Redistribution and Consumption,” Review of Economics and Statistics, May 1955, pp. 149-59.

Tobin, J., “Asset Holdings and Spending Decisions,” AER May 1952, pp. 109-23.

Crockett, Jean, “Income and Asset Effects on Consumption: Aggregate and Cross Section,” and comments by D. B. Suits, in N.B.E.R., Models of Income Determination, pp. 97-136.

Tobin, J., “On the Predictive Value of Consumer Intentions and Attitudes,” The Review of Economics and Statistics, vol. 41, February 1959, pp. 1-11.

 

Optional

Bailey, M. J., “Saving and the Rate of Interest,” Journal of Political Economy, vol. 45, August 1957, pp. 279-305. Reprinted in Landmarks in Political Economy, edited by E. J. Hamilton, A. Rees, and H.G. Johnson (Chicago, 1962), pp. 583-622.

Brown, B., and F. M. Fisher, “Negro-White Savings Differentials and the Modigliani-Brumberg Hypothesis,” Review of Economics and Statistics, vol. 40, February 1958, pp. 79-81.

Brown, E. C., Solow, R. M., Ando, A., and J. Karekan, “Lags in Fiscal and Monetary Policy,” in Commission on Money and Credit, Stabilization Policies (Englewood Cliffs, 1963), pp. 1-165.

Clark, J.M., “Note on Income Redistribution and Investment,” AER, vol. 37, December 1947, p. 931.

Dennison, E. F., “A Note on Private Saving,” Review of Economics and Statistics, August 1958.

Dobrovolsky, S. P., Corporate Income Retention 1915-43 (New York, 1951). (Omit the details.)

Domar, E.D., Essays in the Theory of Economic Growth (New York 1957), pp. 154-67, 195-201.

Ferber, R., “The Accuracy of Aggregate Savings Functions in the Post-War Years,” Review of Economics and Statistics, vol. 37, May 1955, pp. 134-48.

Friedman, M., and G. Becker, “A Statistical Illusion in Judging Keynesian Models,” JPE, vol. 65, February 1957.

Friend, I., and S. Schor, “Who Saves?,” The Review of Economics and Statistics, vol. 41, May 1959, pp. 213-45.

Goldsmith, R. W., A Study of Saving in the United States, three volumes (Princeton, 1952).

Gordon, M. J., “The Optimum Dividend Rate,” presented at the sixth Annual International Meeting of the Institute of Management Sciences, Paris, September 1959. (On library reserve.)

Heller, W. W., Boddy, F. M., and C. L. Nelson, Savings in the Modern Economy, a Symposium (Minneapolis, 1953).

Katona, G., and E. Mueller, Consumer Expectations 1953-56 (Ann Arbor, Michigan, 1956).

Rees, and Johnson, H. G., (Chicago, 1962), pp. 583-622.

Klein, L. R., “The Friedman-Becker Illusion,” JPE, vol. 66, December 1958.

Klein, L. R., (ed.), Contributions of Survey Methods to Economics (New York, 1954).

Morgan, J. N., Consumer Economics (New York, 1955).

Modigliani, F., and R. Brumberg, “Utility Analysis and the Consumption Function: An Interpretation of Cross-Section Data,” in Kurihara, K. K., (ed.), Post Keynesian Economics (New Brunswick, N. J., 1954), pp. 388-436.

Mincer, J., “Employment and Consumption,” Review of Economics and Statistics, vol. 42, February 1960, pp. 20-26.

Zellner, Arnold, “The Short-Run Consumption Function,” Econometrica, (October, 1957).

 

VI. Investment

 

Required

Ackley, Chapter 17.

Keynes, General Theory, Chapters 11, 12.

White, W. H., “Interest Inelasticity of Investment Demand,” AER, vol. 46, September 1956, pp. 565-587.

Knox, “The Acceleration Principle and the Theory of Investment,” Economica, August 1952, pp. 269-97.

Meyer, J., and E. Kuh, The Investment Decision, Chapters 2, 8, 12.

Eisner, R., “Investment: Fact and Fancy,” Jorgenson, D.W., “Capital Theory and Investment Behavior,” Kuh, E., “Theory and Institutions in the Study of Investment Behavior,”: all three in AER, May 1963, pp. 237-268.

Lovell, M.C., “Determinants of Inventory Investment,” in N.B.E.R., Models of Income Determination, pp. 177-216.

Solomon, E., ed., The Management of Corporate Capital, pp. 48-55, 67-73.

Witte, J. G., “The Microfoundations of the Social Investment Function,” Journal of Political Economy, vol. 71, October 1963, pp. 441-56.

 

Optional

Andrews, P.W.S., “Further Inquiry into the Effects of Rates of Interest,” Oxford Economic Papers, February 1940, pp. 32-73.

Brockie, M.D., and A.L. Grey, “The Marginal Efficiency of Capital and Investment Programming,” Economic Journal, vol. 46, December 1956.

Cunningham, N.J., “Business Investment and the Marginal Cost of Funds,” Metroeconomica, vol. 10, August 1958.

Cunningham, N.J., “Business Investment and the Marginal Cost of Funds,” Part II, Metroeconomica, December 1958.

Duesenberry, J., Business Cycles and Economic Growth (New York, 1958), Chapters 4-7.

Ebersole, J.F., “The Influence of Interest Rates,” Harvard Business Review, vol. 17, 1938, pp. 35-39.

Foss, M.F., “Manufacturers’ Inventory and Sales Expectations—A Progress Report on a New Survey,” Survey of Current Business, August 1961.

Foss, M.F., and V. Natrella, “Ten Years’ Experience with Business Investment Anticipations,” Survey of Current Business, January 1957.

Foss, M.F., “Investment Plans and Realizations—Reasons for Differences in Individual Cases,” Survey of Current Business, June 1957.

Friend, I., and J. Bronfenbrenner, “Business Investment Programs and Their Realization,” Survey of Current Business, December 1950.

Grey, A.L., and M.D. Brockie, “The Rate of Interest, Marginal Efficiency of Capital and Net Investment Programming: A Rejoinder,” Economic Journal, June 1959.

Heller, W.W., “The Anatomy of Investment Decisions,” Harvard Business Review, March 1951, pp. 95-103.

Henderson, H.D., “The Significance of the Rate of Interest,” Oxford Economic Papers, October 1938, pp. 1-13.

Hirschleifer, J., “On the Theory of Optimal Investment Decision,” The Journal of Political Economy, vol. 66, August 1958, pp. 329-352. (An excellent but difficult paper.)

James, E., A Reconsideration of the Theoretical Criteria for Optimum Investment Planning (M.I.T. doctoral dissertation 1961).

Lerner, A.P., “On the Marginal Product of Capital and the Marginal Efficiency of Investment,” Journal of Political Economy, vol. 51, February 1953, pp. 1-14. Reprinted in Landmarks in Political Economy edited by E.J. Hamilton, A. Rees, and H.G. Johnson (Chicago, 1962), pp. 538-58.

Lovell, M.C., “Determinants of Inventory Investment,” in Conference on Research in Income and Wealth, Models of Income Determination (Princeton, 1964), vol. 28, pp. 177-232.

Lutz, F.A., and V., The Theory of Investment of the Firm (Princeton, 1951).

Lydall, H.F., “The Impact of the Credit Squeeze on Small and Medium Sized Manufacturing Firms,” Economic Journal, vol. 47, September 1957.

Meade, J.E., and P.W.S. Andrews, “Summary of Replies to Questions on Effects of Interest Rates,” and “Further Inquiry into the Effects of Rates of Interest,” Oxford Economic Papers, No. 1, 1938 and No. 3, 1940.

N.B.E.R., The Quality and Economic Significance of Anticipations Data, A Conference of the Universities—National Bureau Committee for Economic Research (Princeton, 1960).

Penrose, E.T., The Theory of the Growth of the Firm (Oxford, 1959).

Penrose, E.T., “Limits to the Growth and Size of Firms,” AER Papers and Proceedings, vol. 45, May 1955, pp. 531-43.

Pitchford, J.D. and A.J. Hagger, “A Note on the Marginal Efficiency of Capital,” Economic Journal, vol. 48, September 1958, pp. 597-600.

Robinson, J., The Accumulation of Capital (London, 1956). (Wish we had time for it.)

Sayers, R.S., “Business Men and the Terms of Borrowing,” Oxford Economic Papers, February 1940, pp. 23-31.

Spiro, A., “Empirical Research and the Rate of Interest,” Review of Economics and Statistics, vol. 40, February 1958.

Lintner, J., “Corporation Finance: Risk and Investment,” in N.B.E.R., Determinants of Investment Behavior (Robert Ferber editor), pp. 215-54.

Jorgenson, D.W., “The Theory of Investment Behavior,” in N.B.E.R., Determinants of Investment Behavior, pp. 129-55.

Miller, M.H. and F. Modigliani, “Estimates of the Cost of Capital Relevant for Investment Decisions under Uncertainty,” in N.B.E.R., Determinants of Investment Behavior, pp. 179-214.

Miller, M.H. and F. Modigliani, “Reply,” in N.B.E.R., Determinants of Investment Behavior, pp. 260-70.

Lovell, M.C., “Sales Anticipations, Planned Inventory Investment, and Realizations,” in N.B.E.R., Determinants of Investment Behavior, pp. 537-80.

 

Reading List—Fourth Installment
VII. Multiplier and Accelerator

Required

Kahn, R.F., “The Relation of Home Investment to Unemployment,” Economic Journal, 1931. Republished in Hansen and Clemence, Readings in Business Cycles and National Income (New York, 1953), Essay 15.

Readings in Business Cycle Theory, Essays 9-12.

Haavelmo, T., “Multiplier Effects of a Balanced Budget,” Econometrica, 1945, reprinted in Readings in Fiscal Policy, pp. 335-343.

Salant, William A., “Taxes, Income Determination, and the Balanced Budget Theorem,” The Review of Economics and Statistics, May 1957. Reprinted in Gordon and Klein (eds.) A.E.A. Readings in Business Cycles (1965).

Tsiang, S.C., “Accelerator, Theory of the Firm, and the Business Cycle,” Quarterly Journal of Economics, vol. 65, 1951.

 

Optional

Tinbergen, “Statistical Evidence on the Acceleration Principle,” Economica, vol. 5, 1938.

Eisner, R., “Capital Expenditures, Profits, and the Acceleration Principle,” and comments by G.H. Hickman, in Conference on Research in Income and Wealth, Models of Income Determination, (Princeton, 1964), vol. 28, pp. 137-176.

Peston, M.H., “Generalizing the Balanced Budget Multiplier,” and “Comment” by W.A. Salant, The Review of Economics and Statistics (August, 1958).

Bowen, W.G., “The Balanced-Budget Multiplier: A Suggestion for a More General Formulation,” The Review of Economics and Statistics, May 1957.

Goodwin, R.M., “The Multiplier” in Seymour E. Harris, ed., The New Economics (New York, 1947), pp. 482-99.

Chenery, H.B., “Overcapacity and the Acceleration Principle,” Econometrica, vol. 20, January 1952, pp. 1-28.

Caff, J.T., “A Generalization of the Multiplier-Accelerator Model,” The Economic Journal, vol. 69, March 1961, pp. 36-52.

Kuznets, S., “Relation Between Capital Goods and Finished Products in the Business Cycle,” in Economic Essays in Honor of Wesley Clair Mitchell, (New York, 1935).

Knox, A.D. “The Acceleration Principle and the Theory of Investment: A Survey,” Economica, vol. 19, 1952.

Harrod, R.F., Towards a Dynamic Economics (London, 1948).

Hicks, J.R., A Contribution to the Theory of the Trade Cycle (Oxford, 1950).

Goodwin, R.M., “Problems of Trend and Cycle,” Yorkshire Bulletin, vol. 5, August 1953.

Ott, A.E., “The Relation Between the Accelerator and the Capital Output Ratio,” Review of Economic Studies, vol. 25, June 1958.

Minsky, H., “Monetary Systems and Accelerator Models,” American Economic Review, vol. 47, 1957.

Friedman, M. and D. Meiselman, “The Relative Stability of Monetary Velocity and the Investment Multiplier in the United States, 1897-1958,” Stabilization Policies, Commission on Money and Credit (New Jersey, 1963), pp. 165-268.

Hester, D.D., “Keynes and the Quantity Theory: A Comment on the Friedman-Meiselman CMC Paper,” the reply by Friedman and Meiselman, and the rejoinder by Hester, The Review of Economics and Statistics, vol. XLVI, November 1964, pp. 364-377.

 

VIII. Employment and Inflation

Required

Ackley, Chap. XVI.

Bronfenbrenner, M. and F.D. Holzman, “Survey of Inflation Theory,” American Economic Review, LIII (Sept., 1963), pp. 593-661.

Higher Unemployment Rates, 1957-60, “Structural Transformation or Inadequate Demand,” Subcommittee on Economic Statistics of the Joint Economic Committee, Washington, 1961.

Hines, G.G., “Trade Unions and Wage Inflation in the United Kingdom,” R.E. Studies (October 1964).

Killingsworth, C.L., “Automation, Jobs and Manpower,” from Nation’s Manpower Revolution, Hearings before the Subcommittee on Employment and Manpower of the Committee on Labor and Public Welfare, 88th Congress, 1stsession, Washington, D.C., part 5, pp. 1461-1480.

Lipsey, Richard, “The Relation Between Unemployment and the Rate of Change in Money Wage Rates in the United Kingdom, 1862-1957: A Further Analysis,” Economica N.S. 27 (Feb. 1960). Reprinted in Klein and Gordon (eds.), Readings in Business Cycle Theory (1965).

Perry, George L., Unemployment, Money Wage Rates and Inflation (1966).

Phillips, “The Relation Between Unemployment and the Rate of Change of Money Wage Rates,” Economica (Nov., 1958), pp. 283-99.

Samuelson, P.A. and R. Solow, “Analytical Aspects of Anti-Inflation Policy,” American Economic Review (May 1960), pp. 177-94.

Solow, R.M., “The Case Against the Case Against the Guidelines,” in G. Schultz (ed.), Guidelines (1966).

 

Optional

Smithies, A., “The Behavior of Money National Income Under Inflationary Conditions,” Readings in Fiscal Policy, pp. 121-36.

Machlup, F., “Another View of Cost-Push and Demand-.Pull Inflation,” Review of Economics and Statistics, XLII, (May 1960), pp. 125-39.

Galbraith, J.K., “Market Structure and Stabilization Policy,” Review of Economics and Statistics (May 1957), pp. 124-33.

Hicks, J.R., “Economic Foundations of Wage Policy,” Economic Journal, (Sept. 1955), pp. 389-404.

Morton, W.A., “Trade Unionism, Full Employment and Inflation,” American Economic Review, (March 1950), pp. 13-39.

Slichter, S., “Do Wage-Fixing Agreements Have an Inflationary Bias,” American Economic Review, (May 1954), pp. 332-46.

Berman, B., “Alternative Measures of Structural Unemployment,” Employment Policy and the Labor Market, A.M. Ross, ed.

Joint Economic Committee, Higher Unemployment Rates, 1957-60, U.S. 87th Congress.

Galloway, “Labor Mobility, Resource Allocation and Structural Unemployment,” American Economic Review (Sept. 1963), pp. 694-716.

Gordon, R.A., “Has Structural Unemployment Worsened,” Industrial Relations (May 1964), pp. 53-77.

 

Source: Duke University. David M. Rubenstein Rare Book and Manuscript Library. Economists’ Papers Archive. Evsey D. Domar Papers. Box 15, Folder “Macroeconomics. Old Reading Lists”.

______________________

The Theory of Income and Employment
14.451
E. D. Domar [and] J. R. Harris

Midterm Examination
November 30, 1967

(One hour and fifteen minutes)

Please answer all questions. Use a separate book for each question.

  1. (25%) After the discovery that an hour of dancing a day increases a person’s efficiency, a hitherto unemployed dancing teacher was hired (to teach dancing to their employees or themselves) by the following units, one at a time;
    1. A beginning sculptor
    2. The Ford Foundation
    3. Sears, Roebuck & and Co.
    4. The Town of Concord
    5. The Head of the Mafia
    6. The Embassy of South Vietnam in Washington

Disregarding any indirect effects (such as the multiplier), indicate and explain how national income and product and the relevant subdivisions in money and in real terms are affected by this act on the assumption that (1) dancing is really effective, and (2) that it is not. Your reasoning is at least as important as your answer.

  1. (20%) “The Federal Reserve-type index is a poor numerator for the measurement of the Residual (Total Factor Productivity), or of any other productivity.”
    Comment fully.
  2. A visitor to M.I.T. has suggested recently that if the Federal Reserve Board buys bonds in the open market in periods of unemployment, then real output, prices and the interest rate—all three—will increase.
    Are these predictions consistent with those of Patinkin and Keynes? How would their predictions and your own results (you may or may not agree with those sages) be changed under conditions of full employment? Explain fully. (35%)
  3. (20%) A Russian economist once stated that Keynes’ variables were as follows:
Independent variables Dependent variables
1. Propensity to consume 1. Savings
2. Marginal efficiency of capital 2. Investment
3. Rate of interest 3. Level of employment
4. Liquidity preference

Comment. Be specific

Source: Duke University. David M. Rubenstein Rare Book and Manuscript Library. Economists’ Papers Archive. Evsey D. Domar Papers. Box 17, Folder “Macroeconomics. Examinations (1 of 3)”.

______________________

THE THEORY OF INCOME AND EMPLOYMENT
14.451
E. D. Domar [and] J. R. Harris

FINAL EXAMINATION
January 23, 1968

Three Hours

PLEASE ANSWER ALL QUESTIONS. THEY CARRY EQUAL WEIGHTS. USE A SEPARATE BOOK FOR EACH QUESTION.

  1. (A) National Product is defined by the U.S. Department of Commerce as the sum of all final goods (and services), each multiplied by its price.

(B) National Income is defined by it as the sum of all net incomes of certain recipients.

Discuss the following questions:

    1. What is a final good (or service) in (A)? What is the reason for this definition?
    2. What is the rationale for multiplying each good (or service) by its price? What assumptions are implied in this procedure? Are they realistic?
    3. Whose net incomes are aggregated? Why? What is a net income? What assumptions does this procedure imply? Are they realistic?
    4. Could you suggest changes or improvements in the above procedures? Justify them.

 

    1. “A high ratio of depreciation to investment is a sign of old age.”
    2. Why is a special definition of money required in the “Price Flexibility and Employment” problems? What is the definition? What assumptions does it rest on?
    3. “If the Balanced-Budget Multiplier is correct, isn’t Say’s Law also correct?

 

  1. Assume that this country is being threatened by inflation and discuss the pros and cons of the following measures allegedly directed against it. Whenever you can, indicate the positions which several economists whose theories were discussed in the course would take on these measures:
    1. (i) A temporary Federal sales tax on all goods and services, or
      (ii) a permanent tax of the same kind.
    2. (i) A redistribution of income from wages to profits, or
      (ii) a more equal distribution of income.
    3. Setting the rate of growth of labor productivity in each industry as the limit for the rate of increase of wages in that industry.
    4. (i) Remitting domestic taxes on American exports, or
      (ii) a reduction in import duties.
    5. A tax on all capital goods.

 

    1. Define and discuss the applicability to investment decisions of the marginal efficiency of investment (also called marginal efficiency of capital, or the internal rate of return) and the discounted present value. Can they give different ranking of investment projects? Why? Which measure would you use?
    2. What major modifications of investment criteria would be required if the investment was done by the U.S. Government in times of unemployment?
    3. Same, if the investment was done by the government of some underdeveloped country?

 

  1. Attempts to estimate the parameters of an aggregate consumption function for the U.S. have yielded the following results:
    1. Cross-section and short-term series analyses estimate a marginal propensity to consume somewhere in the range of .55-.70, this magnitude being lower than the average propensity to consume.
    2. Long-run time series analyses estimate a marginal propensity to consume equal to the average propensity of about .88.

Compare and contrast the assumptions, rationale and implications of the “Previous Peak Income”, “Permanent Income”, and “Lifetime Cycle” hypotheses, each of which purports to reconcile the above observations.

 

Source: Duke University. David M. Rubenstein Rare Book and Manuscript Library. Economists’ Papers Archive. Evsey D. Domar Papers. Box 17, Folder “Macroeconomics. Final Exams (2 of 3)”.

Image Source: Evsey D. Domar at the MIT Museum legacy website.

Categories
Berkeley Chicago Faculty Regulations Harvard Johns Hopkins M.I.T. Michigan Rochester Stanford Uncategorized Yale

Harvard. Report on the General Examination for an Economics PhD, 1970

 

 

What makes this report on the general examination in the economics PhD program at Harvard particularly valuable is its brief survey of the practice at eight other universities: Yale, MIT, Johns Hopkins, Rochester, Stanford, Berkeley, Michigan, and Chicago. 

_____________________

DRAFT

This draft is distributed in Professor Chenery’s absence to permit discussion at the next Department meeting, January 27, 1970.
Professor Chenery or other members of The Committee might wish to record further comments in preparation [of] a final report.

*  *  *  *  *  *  *  *

HARVARD UNIVERSITY
DEPARTMENT OF ECONOMICS

Cambridge, Massachusetts 02135
January 16, 1970

To: The Department of Economics
From: Committee on Graduate Instruction

REPORT ON THE GENERAL EXAMINATION FOR THE PH.D.

In response to a number of requests from students and faculty, the Committee has reexamined at considerable length the requirements for the General Examination. This report summarizes our general assessment in section I and makes specific recommendations for changes in section II. Some related issues needing further consideration are listed in section III.

Although for the past several years graduate students have criticized various aspects of the generals, the main source of dissatisfaction seems to be with the rigidity of “the system” rather than with any particular aspect of it. We have taken advantage of the fact that the Committee now has three student members to try to understand some of the effects of our present procedures on students’ choices and incentives. We have also tried to strike a better balance between preparation for the general examination and other aspects of a student’s training in his first two years.

As a background for our discussion, the secretary of the Committee compiled a useful summary of the regulations in effect at other leading universities, which is attached.

 

ROLE OF THE GENERAL EXAMINATION

The primary functions [sic] of the General Examination is to evaluate the student’s formal preparation in economics before he proceeds to more advanced phases of teaching and thesis preparation. It also serves as a screening device to weed out weak candidates, as a basis for subsequent recommendations for employers, and as an indirect way of organizing the student’s course work in his first two years. These multiple functions produce much of the debate over requirements at Harvard and elsewhere, since a system that is ideal for one purpose has weaknesses for another.

One of the main criticisms of the existing Harvard system is its psychological impact on the student. The need to satisfy the requirements in all fields within a period of several months inhibits most students from exploring non-required topics until after they have passed the generals. On balance, we are impressed with the desirability of adopting a more flexible timing that will encourage the student to get most of his tool requirements out of the way in the first year and use the second year to explore the fields of his special interest and get some taste of actual research. We have tried to maintain the undoubted benefits of an overall examination, however, as compared to a set of course requirements.

Our survey of other departments shows a significant trend toward breaking down the requirements into separate parts and focusing less on the culminating oral examination. Most departments use the qualifying examination in theory as a device for screening first year students, which also reduces the burden of preparing all fields in the second year. In most departments the minimum proficiency in quantitative techniques and economic history is demonstrated by a satisfactory course grade rather than by inclusions in the general examination. Although we have made our own judgements on these questions, we recommend movement in these directions.

Another consideration which makes greater flexibility desirable is the growing proportion of students who are already well prepared in one or more required fields. For many students, the present system therefore encourages too much review of material they have already covered. We feel that those who are adequately prepared on one of the required fields (theory, quantitative method, history) should have an opportunity to satisfy this requirement in their first year in order to make better use of their time thereafter.

Our recommendations are directed toward achieving greater flexibility in the timing of courses and examinations to allow the student to make more effective use of his time. This should enable many students to get started earlier on their optional fields and to make a better choice of their field of specialization. We do not envision any reduction in the total work done in the first two years or any lowering of standards of performance.

 

SPECIFIC RECOMMENDATIONS

General Principles

  1. The general examination should be separated into four component parts—theory, quantitative method, economic history, and special fields—each of which would be graded separately.
  2. The minimum requirement in quantitative method and economic history should be regarded as a “tool requirement” or “literacy test” as has become the practice in the quantitative field. Students wishing to specialize in these fields may offer them at a higher level as one of their special fields.
  3. The term “general examination” would apply to the oral examination on the special fields. (The question of a general grade on all parts as at present was left open.)
  4. There should be no prescribed timing of the four components, other than the stipulation that the required fields be either completed (or write-off courses in progress) at the time of the oral examination on the special fields. Qualified students would be encouraged to complete one or more requirements in the first year.
  5. Two write-offs should be allowed rather than one.
  6. A subcommittee would be set up for economic history (and retained in theory and quantitative method). The standards and ways of satisfying them in the three required fields should be proposed by the three subcommittees and ratified by the GIC and the Department.

The Theory Requirement

  1. The present coverage (roughly 201a, 201b, 202a) should be retained. The examination would continue to be written.
  2. The examination should be offered two or three times a year. (A straw vote by students showed a preference for June, September and January and a margin for September over January.) Most students would take the examination at the end of their first year—in June or September.

The Quantitative Requirement

  1. The present de facto standard of the written examination should be accepted as the “literacy test”.
  2. The requirement can be met either by the present type of written examination (given twice a year) or by a grade of B+ in 221b or 224a. (It is estimated that roughly 75% would be able to qualify by course examination.)

The Economic History Requirement

  1. The history requirement be made parallel to the quantitative requirement in that:
    1. It can be satisfied by course or special departmental examination.
    2. It can either be offered at a minimum level or at a higher level as a special field.
  2. The minimum requirement would be satisfied by a course grade that would allow a similar proportion to qualify in this way (B+ or A- pending further information).
  3. Alternatives to the present 233 sequence (if any) to be established by the history subcommittee.
  4. Minimum standards in both history and quantitative method could be demonstrated by course examination.

The Requirement in Special Fields

  1. Two special fields would be required as the basis for the oral examination, which would also cover general analytical ability.
  2. Advanced theory, econometrics and economic history would be eligible as special fields, but the first two could not both be included. (In the majority view, one applied field apart from history would be required in order to eliminate the possibility of a candidate offering only the three required fields.)
  3. The candidate would be encouraged (or required?) to submit a research paper to be made part of the subject matter and record of the general examination (He is now “expected” to have presented a paper to a working seminar by the end of his second year.)
  4. The general oral examination would normally be taken at the end of the second year, but could not be taken before the qualifying exams in theory, quantitative and history have been passed (or prospective write-offs are in progress.)

QUESTIONS OF GRADING

  1. Should all examinations be either pass-fail or on a more limited grading scale than at present?
  2. Should the passing standard for the course option in both quantitative methods and history be B+?
  3. Should the four requirements be graded separately or combined (as at present) into an overall grade on the General Examination? (The committee favors first the alternative, but would also require “distinguished” performance in at least one area.)

*  *  *  *  *  *  *  *

Examination Requirements at Other Places

Below I summarize examination requirements at eight other places, including Yale, MIT, Hopkins, Rochester, Stanford, Berkeley, Michigan and Chicago. The main findings of the survey are:

  1. It appears that the massive type of “generals” (where all fields and theory are combined in one session) has almost disappeared. With the exception of Hopkins, all of the above schools seem to settle the theory examination at the end of the first year, with special fields examined at the end of the second year.
  2. Among the schools surveyed, only Yale has a written examination in history. Hopkins, Stanford, Chicago and Berkeley require a course, with “satisfactory” grade. MIT and Rochester have no requirement.
  3. Only Yale gives a written in quantitative aspect of the generals. All the other schools have course requirements (satisfactory grade) only.
  4. Practices vary with regard to number of special fields and type of examination. MIT and Hopkins require three, the others two special fields. Examinations at Yale are oral, at the other places written, in some cases both written and oral. In most places the special field examinations must be taken together, but in some (Rochester, Chicago) they can be separated. Throughout, these special examinations seem to be given by the department, and not merely as course examination.
  5. Some provisions of special interest:
    1. Chicago and Rochester’s second year research paper as part of general examination
    2. Stanford’s requirement for distinction in at least one field.

 

I. Yale

Comprehensive Examination

  1. Written examination in theory and econometrics, usually August or September after first year.
  2. Written examination on economic history; usually late spring of second year.
  3. Oral examination in two applied fields, chosen from six and in general analytical ability; late spring of second year. Given by four examiners. Student excused from general examination in special field courses at end of second year. Oral examination in theory, history, quantitative or field outside economics may be substituted for one of the applied fields if candidate has done year’s course work in applied field “with sufficient distinction”.

History and Quantitative

  1. History—written, end of second year, and option to substitute for one special field.
  2. Quantitative—written, end of first year, and option to substitute for one special field.

Other requirements

  1. Has apparently been dropped.
  2. One course credit of explicit research training, second year.
  3. Dissertation to be completed in fourth year.

 

II. MIT

General examination

  1. General examination in theory consists of two written papers—micro and macro, given in final exam period of first year. May be substituted for final examinations in theory courses.
  2. General examination normally at end of second year. Consists of:
    1. written examinations on three of 12 special fields. These may include advanced theory, econometrics or economic history.
    2. oral examination in the three fields after written.
    3. a fourth field is required but may be written off by B grade in full year course.

History and Quantitative

  1. History—no requirement. May be a special field.
  2. Quantitative—no generals examination. May be a special field.

Other requirements

  1. Two languages

 

III. Johns Hopkins

First Year Oral Examination

A first year oral examination is given in the spring of the first year, covering the fields in which the student has worked during that year.

Comprehensive Examination

Normally taken in spring of second year. Consists of:

  1. Two written examinations in theory, micro and macro.
  2. Three written examinations in special fields, one of which may be outside economics.
  3. Oral examination: Covers theory, special fields, statistics.

History and Quantitative

  1. History—satisfactory work in course.
  2. Statistics—satisfactory work in course.

Other Requirements

  1. One language.
  2. In addition to the departmental special examination, an examination is given by the graduate board, which includes members of other departments.

 

IV. Rochester

Qualifying Examination

  1. Theory and econometrics courses are required but are not part of Qualifying Examination.
  2. Qualifying Examination taken in May of second year. Consists of
    1. Written examination in two fields. These may include mathematical economics and econometrics. Need not be taken simultaneously.
    2. A second year research paper which is to be presented to a departmental seminar at the end of second year.
    3. After (a) and (b) are met, an oral examination in the special fields.

History and Quantitative

  1. Econometrics and mathematical economics requirements (courses), extent depending on fields.
  2. No history requirement.

Other Requirements

  1. Certain distribution requirement.
  2. Language and mathematics.

 

V. Stanford

Comprehensive Examination

  1. Written in micro and macro theory at end of first year. Cover course materials.
  2. Selection of special fields under two plans:
    1. If no minor subject is taken, student chooses four out of ten fields. These may include history, econometrics, mathematical economics. One field may be outside economics.
    2. Student may choose a minor subject (in another department) and choose only one out of the ten special economics fields.

Comprehensive written examinations for each field scheduled annually, usually at close of course sequence. Must show distinction in at least one field.

History and Quantitative

  1. History—Include at least two courses from offerings in economic history, history of thought, comparative economics, development.
  2. Quantitative—Econometrics course required.

Other Requirements

  1. Language or particular quantitative skills.
  2. Two seminars and research papers.

 

VI. Berkeley

Departmental Examination in Theory

  1. Must be passed by end of first year. Students with strong background take it in November of first term, others in June (end of first year).
  2. Written qualifying examinations given in two out of thirteen special fields at end of second year. Examinations given twice a year, must be taken together.
  3. Within one year after written qualifying examinations are completed, student presents himself for oral, based on prospectus (and interim results) of his thesis. General assessment of competence.

History and Quantitative

  1. Course in economic history at 210 level.
  2. Course in statistics at 240 level.

Other Requirements

  1. No language.

 

VII. Michigan

Preliminary Examination

  1. At end of theory courses in micro and macro, an “augmented examination” is given which serves as preliminary examination in theory.
  2. Two fields of specialization are required. One field is satisfied by satisfactory grades in two courses. For the other field a written preliminary examination is required.
  3. After this, oral examination on research topic and surrounding area.

Economic History and Quantitative

  1. No history requirement.
  2. Course requirement in statistics and econometrics.

Other Requirements

  1. No general language requirement.

 

VIII. Chicago

Preliminary Examination

  1. A “course [sic, “core” probably intended] examination” covering micro and macro theory is given twice a year (separate from course examinations) and is usually taken at end of first or middle of second year.
  2. Two special fields are chosen. Written examinations in these fields, separate from course examinations. Need not be taken together.
  3. Student presents a thesis prospectus before thesis seminar, usually in third year. Must pass on this for candidacy.

History and Quantitative

  1. History course required as part of distribution requirements.
  2. Course work in statistics required.

Other Requirements

  1. Math, no languages.

 

Source: John F. Kennedy Presidential Library. John Kenneth Galbraith Papers. Series 5. Harvard University File, 1949-1990. Box 526. Folder “Harvard University Department of Economics: General Correspondence, 1967-1974 (2 of 3)”.

Image Source: Harvard Class Album, 1946.